You are on page 1of 180

RECALLS 3 - NP1

Total points51/100

Situation
You are a nurse caring for patients in an emergency and disaster situation at Brgy. Amolo. The
following questions apply:

1.The emergency and disaster nurse knows that which among the following population
groups have the greatest risk during a disaster?
i. Health care providers
ii. Single parent families
iii.Children
iv. Low income families
*

1/1

a. i, ii
b. i, ii, iii
c. i, iv
d. iii, iv

2.During burn therapy, morphine is primarily administered IV for pain management because
this route:
*

1/1

a. Delays absorption to provide continuous pain relief.


b. Facilitates absorption because absorption from muscles is not dependable.

c. Allows for discontinuance of the medication if respiratory depression develops


d. Avoids causing additional pain from IM injections

Situation
Nurse Cathy is working as a principal investigator on a research project sponsored by their hospital.
She must be knowledgeable about research concepts so that their study results will be successfully
executed. The following questions apply:

3. Nurse Cathy is working as a principal investigator on a research project sponsored by their


hospital. She must be knowledgeable about research concepts so that their study results will
be successfully executed. The following questions apply:
*

1/1

a. Evidence-based practice
b. Research utilization

c. Research dissemination
d. Cochrane Collaboration

4.This refers to the integration of research findings where the practical application is related
to the original research. It starts with a clinical question rather than a research problem.
*

1/1

a. Research utilization
b. Research dissemination
c. Evidence-based practice

d. Cochrane Collaboration

5.You are a nurse and you wanted to study the relationship between post-traumatic growth of
soldiers and social support from their families, what is the independent variable?
*

1/1

a. Social support

b. Post-traumatic growth
c. Soldiers
d. Families
6.Based on the question above, which is the outcome variable?
*

0/1

a. Social support

b. Post-traumatic growth
c. Soldiers
d. Families
Correct answer
b. Post-traumatic growth

7.A research is being conducted to study the the effect of 3 different generations of oral
contraceptives (OC) to the development of myocardial infarction (MI). Which of the
following is the independent variable?
*

1/1

a. Oral contraceptives
b. Myocardial infarction
c. 3 generations of oral contraceptives

d. 3 generations

8.Based on the above situation, does the research need to establish a control group to
determine which among the 3 generations of OC is most likely to cause the development of
MI?
*

1/1

a. Yes. The control group receives no intervention at all.

b. No. The effects of the 3 generations of OC are compared to one another.


c. Yes. The control group receives injectable contraceptive.
d. No. One of the generations of OC serves a comparison group.
9.A nurse is preparing a research proposal wherein she will compute the sample size from a
population of 1, 646 with a margin of error of 5%. What is the computed sample size using
Slovin’s formula?
*

1/1

a. 82.3
b. 329
c. 1, 166
d. 322

10. A nurse research conducts case control study on the association of long-term use of
tamoxifen and breast cancer, and found out an odds ratio of 0.99. The nurse correctly infers
that:
*

1/1

a. 99 cases of breast cancer are attributable to long-term use of tamoxifen.


b. People with breast cancer were 0.99 times more likely to have a history of long-term use of
tamoxifen than those who do not have breast cancer.
c. Risk of breast cancer among those who have history of long-term use of tamoxifen is 0.99 less
compared to those who have no history.

d. 99% of patient who have COPD have history of chronic cigarette smoking.

11. In a research proposal, a nurse mentioned that purposive sampling will be used as a
sampling method. The nurse understands that they will select samples by:
*

1/1

a. Asking early sample members to refer other people who meet the eligibility criteria.
b. Dividing the population into subsets from which elements are selected at random.
c. Using the researcher’s knowledge about the population to hand-pick sample members.

d. Establishing a sampling frame where the sample members are chosen at random.
12. A public health nurse recorded 10 new cases of measles for the months July to
December 2017. Currently, 15 are being treated at the health center since the start of the year.
What is the incidence considering a population of 125,000?
*

1/1

a. 23 per 100, 000 for the year 2017


b. 15 per 100, 000 for July to December 2017
c. 12 per 100, 000 for the year 2017
d. 8 per 100, 000 for July to December 2017

Situation
The community health nurse plays an important role in the community as the primary agent of
information and dissemination on certain infectious diseases. The following questions apply:

13. An important role of the community health nurse in the prevention and control of Dengue
Hemorrhagic-fever includes:
*

1/1

Advising the elimination of vectors by keeping water containers covered


b. Conducting strong health education drives/campaign directed toward proper garbage disposal
c. Explaining to the individuals, families, groups and community the nature of the disease and its
causation.

d. Practicing residual spraying with insecticides

14. Community health nurses should be alert in observing a Dengue suspect. The following is
NOT an indicator for hospitalization of H-fever suspects?
*

1/1

a. Marked anorexia, abdominal pain and vomiting


b. Increasing hematocrit count
c. Cough of 30 days
d. Persistent headache

15. The community health nurse’s primary concern in the immediate control of hemorrhage
among patients with dengue is:
*

1/1

a. Advising low fiber and non-fat diet


b. Providing warmth through light weight covers
c. Observing closely the patient for vital signs leading to shock
d. Keeping the patient at rest.

16. Which of these signs may NOT be REGARDED as a truly positive signs indicative of
Dengue H-fever?
*

1/1

a. Prolonged Bleeding Time


b. Appearance of at least 20 petechiae within 1 cm square

c. Steadily increasing hematocrit count


d. Fall in the platelet count

17. Which of the following is the most important treatment of patients with Dengue H-fever?
*

0/1

a. Give aspirin for fever


b. Replacement of body fluids
c. Avoid unnecessary movement

d. Ice cap over abdomen in case of melena


Correct answer
b. Replacement of body fluids

Situation
The community health nurse of Barangay Jose, Nurse Ian, performed a rapid appraisal of their
community. After assessment, she concludes that there is a high prevalence of alcoholism among their
community's youth. She decided to start a program entitled "PAG-INOM BAWASAN, UPANG
BUHAY AY MADAGDAGAN":

18. During the initial planning of the program, Nurse Ian knows that he can collaborate with
which of the following persons?
1. Chief of police
2. Municipal health officer
3. Guidance counselor of the public high school
4. SK Chairman
5. Parish priest
6. NGO representative
*

0/1

a. All but 1
b. All but 1 & 6
c. All but 5

d. All but 5 & 6


Correct answer
d. All but 5 & 6

19. Nurse Ian knows that his advocacy will best work and will be effective if the following
principle is used as a guide:
*

0/1

a. Self-help
b. Individual therapy
c. Group-help

d. Group therapy
Correct answer
d. Group therapy

20. Nurse Ian makes a plan for the program. He constructs the program aim as:
*
0/1

a. Abstinence from alcohol


b. Moderate withdrawal from alcohol

c. One at a time step towards total alcohol withdrawal


d. Total withdrawal from alcohol
Correct answer
a. Abstinence from alcohol

21. After making a program plan, Nurse Ian decides that a seminar on the legal implications
of substance abuse must be implemented. He knows that the person with the ability to handle
and implement the activity is the:
*

0/1

a. Municipal mayor

b. Guidance counselor
c. Municipal health officer
d. Chief of police
Correct answer
d. Chief of police

22. Before making the program plan, Nurse Ian knows that he must understand the factors
leading to this behavior of the youth. He understands that the person/s with the most
influence on the development of alcoholism among the youth is/are the:
*

0/1

a. Parents

b. Friends
c. Teachers
d. Environment
Correct answer
b. Friends
Situation
Nurse Shakira is a newly hired staff nurse in Polilan General Hospital. She is learning about the
guidelines in their hospital regarding documentation. The following questions apply:

23. The nurse is documenting nursing care for the morning shift. She writes the patient’s
normal findings, the problems, and the routine nursing interventions and evaluation measures
she performed using paragraphs. Which of the following documentation systems did the
nurse use?
*

1/1

a. SOAPIE documentation
b. Narrative charting

c. Problem-oriented medical record


d. Focus charting

24. A nurse is charting her nursing care for a patient in their ward who has been admitted for
20 days already. She documents abnormal and significant findings only. Which
documentation system did she use?
*

0/1

a. Focus charting
b. Problem-oriented medical record

c. Charting by exception
d. Narrative charting
Correct answer
c. Charting by exception

25. A nurse is documenting her patient care using focus charting. Which of the following is
appropriate documentation using focus charting?
*

1/1

a. Data: Administered morphine 4mg IV


b. Focus: Pain
c. Response: Facial grimacing with pain rating of 8/10
d. Action: Rates pain at 1/10.

26. Most agencies have specific policies about telephone orders. Which of the following is
not true about telephone orders?
*

0/1

a. It is the physician’s responsibility to sign the order within 24 hours.


b. While the physician gives the order, the nurse listens carefully and writes order.
c. Orders that seem inappropriate must be questioned.
d. The order is transcribed to the doctor’s order sheet

Correct answer
b. While the physician gives the order, the nurse listens carefully and writes order.

Situation
As a community health nurse, it is our primary task to ensure that every child is well equipped with
the necessary materials against known infections. You are handling Barangay Urasan in Amorsolo,
Pagadian with a total population of 3,200. The following questions apply:

27. It was noted that there were 700 infants in the area. How many patients can be computed
as part of the eligible population for vaccine requirement?
*

1/1

a. 20 patients
b. 19 patients

c. 18 patients
d. 17 patients

28. Given the eligible population, what would be the computed vaccine requirement for the
year for DPT?
*

0/1
a. 100 vaccines
b. 90 vaccines
c. 95 vaccines
d. 96 vaccines

Correct answer
c. 95 vaccines

29. A group of pregnant women was also noted to be of low immunization to Tetanus. The
total population of all pregnant women were 368. How many were eligible?
*

1/1

a. 13 patients

b. 14 patients
c. 15 patients
d. 16 patients

30. Given the eligible population, what would be the computed vaccine requirement for the
year for Tetanus?
*

0/1

a. 110 vaccines
b. 108 vaccines
c. 105 vaccines

d. 100 vaccines
Correct answer
b. 108 vaccines

31. What is the wastage multiplier of Tetanus given that an ampule can give 20 doses and
that the number of doses used is 3?
*
0/1

a. 7
b. 8

c. 9
d. 10
Correct answer
a. 7

Situation
You are the public health nurse of Barangay Panungyan. You have just finished implementing the
programs for the identified community health problems last month. You are now preparing to check if
the target goals and objectives were met. The following questions apply.

32. Evaluation involves which of the following processes?


*

1/1

a. Observation
b. Exploration
c. Measurement
d. A and C

33. Which of the following phases of the PRECEDE-PROCEED Model correspond to the
evaluation if the objectives and sub-objectives have been met?
*

0/1

a. Impact Evaluation
b. Process Evaluation
c. Outcome Evaluation

d. Effect Evaluation
Correct answer
a. Impact Evaluation
34. Which of the following is evaluated by the outcome evaluation?
*

1/1

a. Strategy Objectives
b. Objectives
c. Goal

d. Sub-objectives

35. Which of the following evaluation indicators is described by the statement, “what
proportion of those who need something are actually receiving it”?
*

1/1

a. Accessibility
b. Efficiency
c. Coverage

d. Effort

36. What is the ultimate goal of community health nursing?


*

0/1

a. Community competence
b. Community self-actualization
c. Community organization
d. Community integration

Correct answer
a. Community competence

Situation
Mr. Dencio, a lawyer, has been under medical treatment for multiple gastric ulcer for the last 5 years
and has been advised surgery by his doctor.
37. The patient has to undergo a Billroth II surgical operation. Which of the following is
done in this procedure?
*

0/1

a. A partial removal of the upper 1/3 of the stomach and anastomosis to lower esophageal sphincter.
b. Half of stomach is removed. Entire jejunum is excluded from GI tract. Duodenum is disconnected.

c. A partial removal of the distal 2/3 of the stomach and anastomosis of the stump to the duodenum
d. A partial removal of the distal 2/3 of the stomach and anastomosis of the stump to the jejunum
Correct answer
d. A partial removal of the distal 2/3 of the stomach and anastomosis of the stump to the jejunum

38. Prior to surgery, you were asked by Mr. Dencio why atropine sulfate is ordered, your
BEST response is:
*

0/1

a. "Anesthetic agents can increase mucus secretion and atropine will counteract this effect"
b. “Atropine can decrease the rate of forming stools and chances of defecating during the surgery.”
c. “It will help you relax and keep you from feeling the pain from the procedure.”

d. “It will kill bacteria in your intestines, decreasing possibility of infection.”


Correct answer
a. "Anesthetic agents can increase mucus secretion and atropine will counteract this effect"

39. You just finished your pre-operative teaching to Mr. Dencio. Which of the following
statements indicates that further teaching is needed?
*

0/1

a. “I will place my hand or a pillow on my abdomen whenever I have to cough or exert force.”
b. "I can ambulate as soon as I am transferred to the recovery room to prevent blood clot in my legs.”
c. “I must lie down for 30 minutes after eating.”

d. “I should be eating more amount of fried chicken than a bowl of porridge.”


Correct answer
a. “I will place my hand or a pillow on my abdomen whenever I have to cough or exert force.”

40. Which of the following assessment would be your indication that Mr. Dencio has
recovered from anesthesia. A sense of?
*

1/1

a. Paresthesia in the legs


b. Detachment
c. Feeling of warmth in the room

d. Humor

41. After Billroth II surgery, Mr. Dencio reported that he was having diarrhea after meals.
What should the nurse recommend to prevent this condition?
*

1/1

a. Consume 3 meals a day


b. Lie flat while eating meals.
c. Avoid taking fluids.

d. Sit in upright position after meals.

Situation
Nurse Dora handles various patients with elimination problems. She always performs her best nursing
care to these patients.

42. Nurse Dora teaches an incontinent patient bowel training. Which among the following
would Dora not include in her health teaching?
*

0/1

a. Opioids are avoided to prevent decrease in bowel peristalsis.


b. Regular exercise is encouraged.
c. Leaning backward at the hips while sitting on the toilet will help stimulate colon emptying.
d. A hot tea or fruit juice may be taken before the patient defecates.
Correct answer
c. Leaning backward at the hips while sitting on the toilet will help stimulate colon emptying.

43. An enema to provide relief from gaseous distention is prescribed to a patent. Dora knows
that this is a/an:
*

0/1

a. Oil retention enema


b. Carminative enema
c. Soapsuds enema
d. Fleet enema

Correct answer
b. Carminative enema

44. After inserting a nasogastric tube, Dora checks its placement by measuring the pH of the
aspirate. Dora knows that proper placement has likely been made if the aspirate has a pH of:
*

1/1

a. pH 3.0

b. pH 7.0
c. pH 9.0
d. pH 7.4

Situation
An infant was born 12 hours ago. Nurse Katrina has been instructed to collect blood by heel stick for
newborn screening (NBS) for congenital hypothyroidism before the baby will be discharged.

45. Nurse Katrina questions the order of the Pediatrician because 12 hours after birth in not
the optimum time to collect this specimen for which reason?
*

1/1

a. There is an immediate rise of Thyroid stimulating hormone after birth.


b. The baby needs to digest the formula before a blood sample can be taken.
c. A thyroid scan should be done first.
d. At 24 hours, the T4 level will be extremely high

46. At 4 months, the child was diagnosed with hypothyroidism. The mother asks symptoms
consistent with the diagnosis. Select those that apply:
*

0/1

a. 1,4 and 5

b. 3, 4 and 5
c. 1, 2 and 3
d. 2, 3 and 4
Correct answer
b. 3, 4 and 5

47. What will be the PRIORITY Nursing Diagnosis for this child?
*

0/1

a. High risk for ineffective health maintenance.


b. Hyperthermia
c. Imbalance Nutrition more than the body requirements
d. Altered oral mucus membrane

Correct answer
a. High risk for ineffective health maintenance.

48. If not diagnosed early, irreversible damage can occur to what major organ of the body?
*

0/1

a. Brain
b. Gastrointestinal tract
c. Liver
d. Thyroid gland

Correct answer
a. Brain

49. Thyroid hormone replacement to eliminate signs of Hypothyroidism is the lifetime


management. Prior to administration to such drug, the nurse will check which of the
following:
*

1/1

a. Blood pressure
b. Respiratory rate
c. Temperature
d. Pulse rate

Situation
Municipality “A” made it to a local newspaper headline because of the occurrence of some unusual
events in just 10 days: 2 pregnant woman died giving birth at home; 2 children died of dehydration
and 2 elderly died of pneumonia. The mayor of the town called for a meeting of the municipal health
office personnel. Public Health Nurse Maria was assigned to make the action plan to prevent another
similar event to happen.

50. Which of the following could have been the TOP contributors to the situation? Select all
that apply.
1. Postponement of people in seeking medical care
2. Adherence to cultural practices and beliefs
3. Lack of ambulance and drivers
4. Delay in receiving appropriate and adequate care
*

1/1

a. 2 and 3
b. 1 and 2
c. 1 and 4
51. Nurse Maria presented the plan in a community assembly for acceptance and approval.
Which of the following is the MOST important criterion for the plan’s acceptance and
approval manifested by the people?
*

0/1

a. Tasking the community people


b. Volunteerism and willingness of the community people
c. Presence of the municipal health officer in the assembly
d. Huge attendance in the community assembly

Correct answer
b. Volunteerism and willingness of the community people

Situation
The family is the unit of service in Community Health Nursing (CHN). Public Health Nurse Mina
takes care of the Cruz family. Emman works as janitor in the municipal hall. His wife Lynn tends a
small variety store in front of their house. Their children are Llonel (6 yrs. old), Everly (5 yrs. old),
Jam (4 years old) and Manilyn (3 yrs. old).

52.Based on the characteristics of the Cruz family, Nurse Mina can easily categorize which of
the family’s developmental stage?
*

1/1

a. Families with pre-school children

b. Family with school age children


c. Beginning family
d. Early child-bearing

53. In this particular stage, which of the following is a most basic concern?
*

1/1

a. Emergence of harmony in marital and in-law relations.


b. Child rearing
c. The couple shifts to adult social interests.
d. Balancing time and energy to meet the demands of work and family.

Situation
Mr. and Mrs. Salungkit have eight children who are all boys. They would like to be counselled on
family planning methods by Nurse Ram.

54. Nurse Ram explains the two forms of hormonal contraception which are the pill or the
Oral Contraceptive (OC) or the combination of Oral Contraceptive (COC). The COC’s
compositions and the corresponding actions are, select the TWO correct answers.

1. Estrogen acts to suppress the Follicle Stimulating Hormone (FSH) and Luteinizing
Hormone (LH), thus suppressing ovulation.
2. Progesterone complements the action of estrogen causing the decrease in the permeability
of cervical mucus, thereby limiting sperm motility and access to ova.
3. Estrogen acts to increase FHS and LH, thus suppressing ovulation.
4. Progesterone when combined with estrogen causes the increase in the permeability of
cervical mucus thereby decreasing the possibility of tubal transport and implantation.
*

1/1

a. 3 and 4
b. 1 and 4
c. 1 and 2

d. 2 and 3

55. Which of the following are the side effects of COC’s? Select the CORRECT answers.
1. Weight gain
2. Headache
3. Depression
4. Breast tenderness
*

0/1

a. 3 and 4
b. 1 and 2
c. 1, 2, 4
d. 1, 2, 3, 4

Correct answer
c. 1, 2, 4

56. Nurse Ram advises that the BEST way to lessen the side effects of COC’s is by
__________.
*

0/1

a. Changing the routine activities


b. Consulting the physician for the side effects

c. Not minding it because it will just subside


d. Using different brands of contraceptives
Correct answer
a. Changing the routine activities

57. Mrs. Salungkit expressed fear in choosing OC or COCs. She asks about the advantages of
using chemical barriers as methods of contraception. Which of the following are the
advantages of this type of contraception?
1. It interferes with sexual enjoyment
2. When used with another contraceptive, they increase the other method’s effectiveness
3. Various preparations are available in the market
4. They can be purchased without a prescription
*

0/1

a. 1 and 2
b. 3 and 4
c. 1, 2, 3

d. 2, 3, 4
Correct answer
d. 2, 3, 4

Situation
Lem, Kid, Emma, Gliz, and Trix are close friends because they are almost of the same age (8-9 years
old), orphans, and out-of-school youths. Nurse Cess from the Department of Social Welfare and
Development (DSWD) is tasked to attend to them.
58. What could be the number ONE health threat for these children?
*

0/1

a. Infection
b. Depression
c. Malnutrition

d. Injury
Correct answer
d. Injury

59. What should be the PRIMARY goal of Nurse Cess for these street children? To
___________.
*

0/1

a. Change the behavior of the children


b. Analyze the information gathered

c. Locate the family of the children


d. Elicit important data about the family
Correct answer
a. Change the behavior of the children

Situation
Lyka, 20 years old, G3P4, has just delivered to a baby boy one hour ago. Nurse Hanz is the nurse
assigned to the newborn.

60. Which of the following senses are already developed at birth? Select those that apply.
1. Hearing.
2. Vision.
3. Taste.
4. Smell.
*

0/1

a. 1, 2, 3, 4
b. 1, 3, 4
c. 1, 2
d. 1, 2, 3
Correct answer
a. 1, 2, 3, 4

61. One day, Baby Boy did not want to suck from the mother’s breast. The doctor ordered
intravenous fluids. Which among these complications will Nurse Hanz anticipate to occur
ALMOST IMMEDIATELY?
*

1/1

a. Cellulitis
b. Infiltration

c. Phlebitis
d. Burns

62. Later, the pediatrician orders Baby Boy to undergo exchange blood transfusion because
of ABO blood incompatibility. What should be the BEST equipment to use in as much as
50mL of blood is ordered?
*

0/1

a. Syringe pump
b. Soluset with microdip

c. Pulse oximeter
d. Infusion pump
Correct answer
c. Pulse oximeter

63. Pediatric patients are prone to falls from hospital beds which may result to additional cost
on the part of the hospital. Which safety measures should be instituted to prevent such
incidents?
1. Restraining them
2. Having their mothers or any significant adult with them
3. Advising watchers to put up the bedrails at all times
4. Telling them they will be given injection if they will get out of bed alone
*

1/1

a. 3 and 4
b. 2 and 3

c. 1 and 2
d. 1 and 4

64. As a young manager, Nicka knows that conflict occurs in any organization. Which of the
following statements regarding conflict is NOT true?
*

0/1

a. It may result to poor performance of the staff.


b. It can be destructive; hence, it should not reach the highest level.
c. It is not beneficial; hence, it should be prevented at all times.
d. It may create a new leader from among the staff.

Correct answer
c. It is not beneficial; hence, it should be prevented at all times.

65. Which of the following is the BEST action that she must take?
*

1/1

a. Quit her job and look for another employment.


b. Disregard what she feels and continue to work independently.
c. Identify the source of conflict and understand the source of friction.

d. Seek help from the Director of Nursing.

66. Which of the following nursing strategies would be most likely encouraging an ill child to
eat?
*

1/1

a. Feed the child to be sure she eats all the food on the tray
b. Take the child to the play room for all meals.
c. Offer the child food and fluids she likes.

d. Withhold dessert until intake is adequate

67. Which of the following observations of a mother who had a healthy baby 48 hours ago
would alert a nurse’s concern about the mother’s attachment with the newborn?
*

0/1

a. She expresses difficulty in finding a name for the new born.


b. She requests that the father change the newborn’s diaper
c. She stares out at the window while feeding the newborn.
d. Asking repeated questions about infant care.

Correct answer
c. She stares out at the window while feeding the newborn.

Situation
Nurse Jayson is at the Labor and Delivery Room caring for 5 pregnant women experiencing labor in
their various stages.

68. Mrs. Ahya is a primiparous woman. The nurse expects that the average range of her labor
would be?
*

1/1

a. 8 – 14 hours
b. 14 – 20 hours

c. 1 – 8 hours
d. 20 – 24 hours
69. Mrs. Regine is a multiparous woman. She has been on labor for the past 16 hours and she
is still 6cm dilated. This can be noted as?
*

1/1

a. Normal Labor
b. Extended Labor
c. Prolonged Labor

d. Precipitous Labor

70. Which among the following stages of labor is deemed to be the most critical stage?
*

0/1

a. 1st Stage
b. 2nd Stage
c. 3rd Stage

d. 4th Stage
Correct answer
d. 4th Stage

Situation
At 32 weeks gestation, patient Marjorie, a 16-year-old primigravid, 5 feet tall, gained a total of 20
pounds, with one-pound gain in the last 2 weeks. Urinalysis reveals negative for glucose and a trace of
protein. She came to the clinic for consultation.

71. Which will BEST describe Marjorie’s risk for preeclampsia?


*

0/1

a. Proteinuria
b. Short stature
c. Total weight gain

d. Adolescent age group


Correct answer
d. Adolescent age group

72. After instructing Marjorie how to keep a record of daily fetal movement counting
(DFMC) at home, the nurse determines that the teaching has been effective when Marjorie
says she will count the number of times the baby moves during which of the spans?
*

0/1

a. 45-minute period after lunch each day

b. 12-hour period each day


c. 30-minute period 3 times a day
d. 1-hour period each day
Correct answer
d. 1-hour period each day

73. When teaching Marjorie about nutritional needs, the nurse should emphasize
___________.
*

0/1

a. High residue diet

b. Low sodium diet


c. Regular diet
d. High protein diet
Correct answer
c. Regular diet

74. The nurse mentions some of the conditions associated with preeclampsia. Which among
the feedbacks by Marjorie to the nurse would warrant further explanation?
*

0/1

a. Iron deficiency
b. Physical disability
c. Multiple pregnancy
d. Diabetes mellitus
Correct answer
b. Physical disability

75. Marjorie was brought to the labor room for magnesium sulfate medication. The
PRIMARY purpose for its administration is to:
*

1/1

a. Increase the central nervous system’s response to stimuli


b. Increase calcium absorption by the muscles
c. Decrease neuromuscular irritability

d. Reduce peripheral vascular resistance to lower blood pressure

Situation
The pediatric nurse is preparing necessary research for her case presentation at the Pediatric Nurses
Association of the Philippines. Her case is an 8-month-old patient with phenylketonuria, the following
questions apply.

76. What type of inheritance does PKU belong?


*

1/1

a. Autosomal Dominant
b. Autosomal Recessive

c. Autosomal Dormant
d. Autosomal Reflexive

77. The nurse was asked what milk should be given to the child. What would be her best
answer?
*

0/1

a. You can buy Bear-brand or Nido for the patient.


b. You need to buy Lofenalac.
c. You need to buy phenylalanine-rich milk.

d. Any would do.


Correct answer
b. You need to buy Lofenalac.

78. As part of her long-term nursing care, she was preparing some of her health teaching
plans for the patient. Which among the following should be included in her teaching plan?
*

1/1

a. Patient should need to have rehabilitation of muscles.


b. Patient should need to have monthly consult with the pediatrician for signs and symptoms of atopic
dermatitis.

c. Patient should do outdoor activities with ease and flexibility.


d. Patient should need to bring oxygen masks and tanks since the lungs may be compromised.

79. Which among the following is known to be the end point of a patient with PKU if let
untreated?
*

1/1

a. Death
b. Mental Retardation

c. Blindness
d. Lethargy

80. Episiotomy is usually indicated for which of the following purposes?


*

1/1

a. To prevents distention of the bladder.


b. To relieve pressure on the fetal head.
c. To aid in contraction of the uterus following delivery.
d. Done primarily for the physician’s benefit.

81. A 39-year-old at 37 weeks gestation is admitted to the hospital with complaints of vaginal
bleeding following the use of cocaine 1 hour earlier. Which complication is most likely
causing the client’s complaint of vaginal bleeding?
*

0/1

a. Placenta previa
b. Abruptio placentae
c. Ectopic pregnancy
d. Spontaneous abortion

Correct answer
b. Abruptio placentae

82. A pregnant woman arrives at the emergency department (ED) with abruptio placentae at
34 weeks’ gestation. She’s at risk for which of the following blood dyscrasias?
*

1/1

a. Thrombocytopenia.
b. Idiopathic thrombocytopenic purpura (ITP).
c. Disseminated intravascular coagulation (DIC).

d. Heparin-associated thrombosis and thrombocytopenia (HATT).

83. A maternity nurse is caring for a client with abruptio placentae and is monitoring the
client for disseminated intravascular coagulopathy. Which assessment finding is least likely
to be associated with disseminated intravascular coagulation?
*

0/1

a. Swelling of the calf in one leg


b. Prolonged clotting times
c. Decreased platelet count
d. Petechiae, oozing from injection sites, and hematuria
Correct answer
a. Swelling of the calf in one leg

84. A nurse is assessing a pregnant client in the 2nd trimester of pregnancy who was admitted
to the maternity unit with a suspected diagnosis of abruptio placentae. Which of the following
assessment findings would the nurse expect to note if this condition is present?
*

0/1

a. Absence of abdominal pain


b. A soft abdomen
c. Uterine tenderness/pain
d. Painless, bright red vaginal bleeding

Correct answer
c. Uterine tenderness/pain

85. An ultrasound is performed on a client at term gestation that is experiencing moderate


vaginal bleeding. The results of the ultrasound indicate that an abruptio placentae is present.
Based on these findings, the nurse would prepare the client for:
*

0/1

a. Complete bed rest for the remainder of the pregnancy

b. Delivery of the fetus


c. Strict monitoring of intake and output
d. The need for weekly monitoring of coagulation studies until the time of delivery
Correct answer
b. Delivery of the fetus

Situation
The Ebola Virus causes an acute serious illness which is often fatal if untreated. Ebola Virus first
appeared in 1976 in two simultaneous outbreaks, one was in Sudan and one was in Congo. The
outbreak which occurred in 2014 in West Africa was the largest and most complex Ebola outbreak
since the virus was first discovered.

86. The following are signs and symptoms of the Ebola Virus Disease. Select all that apply:
(i) Sudden onset of fever
(ii) Splenomegaly
(iii) Fatigue and muscle pain
(iv) Bleeding (Internal and external)
(v) Tinnitus
(vi) Decreased level of consciousness
*

1/1

a. i, iii, iv, v
b. ii, iv, v, vi
c. iii, iv, v, vi
d. i, iii, iv, vi

87. The following basic nursing interventions can aid increase the chances of survival
EXCEPT:
*

0/1

a. Provide symptomatic care.


b. Strictly follow the schedule for the prescribed antibiotics.
c. Administer oral or intravenous fluids, as ordered.
d. Monitor oxygen saturation and blood pressure.

Correct answer
b. Strictly follow the schedule for the prescribed antibiotics.

Situation
Nurse Kevin is reviewing concepts and practicing skills on intravenous transfusions.

88. Nurse Kevin is selecting a vein for an intravenous transfusion in a 62-year-old patient.
He wants to dilate and distend the veins for easier insertion. He does the following methods,
except:
*
0/1

a. Tapping the selected vein multiple times


b. Placing extremity in a dependent position if there are no contraindications
c. Applying warm washcloth on to the extremity for several minutes
d. Stroking the extremity from distal to proximal below his desired site

Correct answer
a. Tapping the selected vein multiple times

89. Which among the following catheters is used for less than 24 hours?
*

1/1

a. Butterfly needle

b. Midline catheter
c. Plastic indwelling cannula
d. Angiocath

90. How often should Nurse Kevin observe a patient with intravenous infusions?
*

1/1

a. Every 2 hours

b. Every 6 hours
c. Every 8 hours
d. Every 12 hours

91. Six hours after initiating an IV line in a patient, Nurse Kevin noticed that the IV site was
red and warm. The patient complained that it was painful. Nurse Daniel would suspect which
of the following?
*

1/1
a. Phlebitis

b. Extravasation
c. Infiltration
d. Clot formation

92. What should be the size/gauge of the needle Nurse Kevin would use in administering IV
infusions for trauma patients?
*

1/1

a. 18

b. 22
c. 20
d. 24

Situation
The community health nurse in Sta. Rosa Health Center is teaching a group of women who are
consulting in the health facility. The following questions apply:

93. A patient 3 days postpartum asks the nurse when her menses will return considering that
she is doing exclusive breastfeeding. The nurse’s best response is:
*

1/1

a. 5-6 weeks
b. 6-8 weeks
c. 4-6 months

d. 1-2 months

94. You are a nurse teaching a female client about congenital rubella. Which of the
following is not part of your teaching?
*

0/1

a. The vaccine is not contraindicated in women who are breastfeeding.


b. Congenital rubella can be prevented by receiving rubella vaccine within the 1st trimester.
c. When the vaccine is given immediately after the delivery, the effects may be delayed for up to 21
days.

d. Congenital rubella syndrome is characterized by deafness, eye problems, brain defects, and patent
ductus arteriosus.
Correct answer
b. Congenital rubella can be prevented by receiving rubella vaccine within the 1st trimester.

95. A couple approached the nurse in the health center with plans to get pregnant. The nurse
will advise the couple with basis on the knowledge that the egg cell lasts for up to:
*

0/1

a. 24-48 hours
b. 6-12 hours
c. 12-24 hours

d. 48-72 hours
Correct answer
a. 24-48 hours

96. Which of the following is true about progesterone?


*

0/1

a. It prepares the endometrium to support implantation and growth of fertilized egg.


b. Its release is stimulated by the follicle-stimulating hormone.
c. It maintains and supports continued growth of endometrium.

d. Progesterone levels decrease when Gn-RH increases.


Correct answer
d. Progesterone levels decrease when Gn-RH increases.

97. The following are correct positioning and latching when the baby is being breastfed
except:
*

0/1

a. Baby latches to the entire areola.

b. Head, shoulder, and hips are aligned.


c. The lower lip is turned inwards.
d. Chin touches the breast.
Correct answer
c. The lower lip is turned inwards.

98. A pregnant mother who was diagnosed as having diabetes mellitus Type 1 is already at 24
weeks’ gestation. Which of the following statements by the nurse is relevant for the client?
*

0/1

a. You need to have optimal glycemic control to keep your blood sugar low.

b. Your baby may reach a fetal weight of 3.7kg upon delivery.


c. Hyperglycemia is the most common complication for your baby when he is delivered.
d. Take the oral antihyperglycemics to be prescribed to you strictly.
Correct answer
b. Your baby may reach a fetal weight of 3.7kg upon delivery.

99. The mother consulted to Sta. Rosa Health Center as she suspects that she is pregnant
after having no menstruation this month. She told the nurse that she had her menses last
February 14, 2018. The mother asked when she will deliver the baby. The nurse correctly
replies:
*

1/1

a. December 3, 2018
b. October 29, 2018
c. November 14, 2018
d. November 21, 2018
100. Which of the following is the FIRST step that Nurse X must consider in preparing a
health teaching plan? To know _____________.
*

1/1

a. Certain factors that may affect the client’s learning.


b. The client’s personal values and expectations
c. The client’s age and educational status
d. The postpartum client’s needs

RECALLS 3 - NP2
Total points50/100

Situation
Lovely is a newly hired nurse at the Neonatal Intensive Care Unit (NICU). She received an
endorsement that Baby Boy A and Baby Girl B are ready for discharge. At 5:15 in the afternoon she
gave the babies to their mothers. Two days after, the mother of “Baby Boy A” came back to complain
that she got the wrong baby because she got a baby girl instead of a boy.

1. Which of the following should Lovely, being professional nurse, understand as the
primary outcome should she be proven guilty of gross incompetence?
*

1/1

a. That she can be delisted as member of the APO


b. That she can be terminated from work by her employer
c. That she can be charged in the civil court
d. That her license to practice nursing can be suspended or revoked for a period of time by the PRC

2. Which among the statements are TRUE about health promotion? It is the process of
__________.
1. Enabling individuals and communities to increase their abilities to control and improve
their health
2. Enhancing the individuals physical, psychologic and spiritual well-being
3. Engaging in correcting the people’s health behavior
4. Preventing disease from occurring and treating it when it occurs
*

0/1

a. 1 only

b. 3 and 4
c. 2 only
d. 1 and 2
Correct answer
d. 1 and 2

Situation
Mrs. Fabricon, 28 years old, has been married for 4 years. She postponed pregnancy to pursue her
career as a lawyer. One day, she went to see an obstetrician because of vaginal spotting and to find
ways of getting her pregnant. After a series of diagnostic examinations, she was found to have
endometriosis.

3. The nurse performs an assessment on Mrs. Fabricon. Which of the findings would be
indicative of endometriosis? Select all that apply.
1. Spotting after intercourse
2. Persistent dysmenorrhea
3. Dyspareunia
4. Menorrhagia
*

1/1

a. 1, 4
b. 3, 4
c. 1, 2, 3
d. 2, 3, 4

4. The physician prescribes Danazol (Danocrine) for Mrs. Fabricon. The nurse should tell
her that while taking this drug she will experience all these symptoms, EXCEPT
____________.
*

0/1
a. Anovulation

b. Weight gain and edema


c. Diminished menstrual flow
d. Cessation of her menses
Correct answer
c. Diminished menstrual flow

5. What is the rationale behind endometriosis interfering with fertility? It is because


____________.
*

1/1

a. Of pressure on the pituitary that leads to decreased FHS levels.


b. Of total blockage of endometrial tissues

c. The uterine cervix becomes inflamed and swollen


d. The ovaries stop producing adequate estrogen

Situation
The nursing director assigned Glenn in the orthopedic ward to care for patients with mobility
problems.

6. Glenn Mapalo is teaching a patient with crutches how to go up the stairs. Which among
the following should he instruct the patient with?
*

0/1

a. When moving the affected leg up, the body weight should be on the crutches.
b. The unaffected leg goes up the step after the affected leg.

c. Patient should keep his feet together at the bottom of the stairs
d. The crutches and the affected leg go up the step together.
Correct answer
d. The crutches and the affected leg go up the step together.
7. Glenn is assisted by another staff nurse in lifting a patient. Which among the following
actions will prevent injury to Glenn?
*

0/1

a. Keeping the feet wide apart and knees extended

b. Positioning himself close to the patient


c. Using the muscles of the back in lifting
d. Rotating at the waist in lifting
Correct answer
b. Positioning himself close to the patient

8. Nurse Glenn asks two more nurses to help him logroll a patient. Where should Glenn
place the patient’s arms prior to logrolling?
*

1/1

a. Along the sides


b. Crossed on the chest

c. Overhead, extended
d. Behind the hips

9. In applying restraints, Glenn Mapalo should employ which type of ie?


*

0/1

a. Half-bow
b. Surgical tie
c. Double knot

d. Two-throw knot
Correct answer
a. Half-bow

Situation
One of the legal responsibilities of nurses is to ensure proper documentation of patient care.

10. Julius, a staff nurse in the medical unit, understands that his charting must consist of the
following information excluding:
*

1/1

a. Patient medical diagnosis

b. Information about the patient's health status


c. Relevant health data
d. Procedures performed

11. Julius shares to a Dagupan Hospital staff nurse the importance of proper documentation.
This includes:
1. Reflects quality and timeliness of nursing care
2. Evidence of care provided by doctors and nurses
3. Facilitates interprofessional communications
4. Utilized as a legal document in a court litigation
5. Holds vital information about the patient
*

1/1

a. All but 1, 2 & 3


b. All but 5
c. All but 4
d. All of the above

12. What should be the size/gauge of the needle Nurse Kevin would use in administering IV
infusions for trauma patients?
*

1/1

a. 18

b. 22
c. 20
d. 24

Situation
An Obstetric Nurse is currently handling patients who are undergoing various pregnant clients for
certain laboratory and diagnostic tests. The following questions apply.

13. The nurse is providing instructions to a pregnant client who is scheduled for an
amniocentesis. What instruction should the nurse provide?
*

1/1

a. Strict bed rest is required after the procedure.


b. Hospitalization is necessary for 24 hours after the procedure.
c. An informed consent needs to be signed before the procedure.

d. A fever is expected after the procedure because of the trauma to the abdomen.

14. The nurse asks about a pregnant client who is in the first trimester calls the nurse at a
health care clinic and reports that she has noticed a thin, colorless vaginal drainage. The nurse
should make which statement to the client?
*

1/1

a. “Come to the clinic immediately.”


b. “The vaginal discharge may be bothersome, but is a normal occurrence.”

c. “Report to the emergency department at the maternity center immediately.”


d. “Use tampons if the discharge is bothersome, but to be sure to change the tampons every 2 hours.”

15. The nurse has performed a nonstress test on a pregnant client and is reviewing the fetal
monitor strip. The nurse interprets the test as reactive. How should the nurse document this
finding?
*

1/1

a. Normal

b. Abnormal
c. The need for further evaluation
d. That findings were difficult to interpret

Situation
Karla is a nurse in the maternity unit. Here, she takes care of gravidomorbid clients. It is essential to
know the normal and abnormal changes in pregnancy in order to deliver the best nursing care. The
following questions apply.

16. During the night shift, a woman is hospitalized for the treatment of severe preeclampsia.
Which of the following represents an unusual finding for this condition?
*

1/1

a. Convulsions.

b. Blood pressure 160/100.


c. Proteinuria 41.
d. Generalized edema.

17. As a nurse in the maternity unit, Charm knows that the action of hormones during
pregnancy affect the body by?
*

1/1

a. Raising resistance to insulin.

b. Blocking the release of insulin from the pancreas.


c. Preventing the liver from metabolizing glycogen.
d. Enhancing the conversion of food to glucose.

18. A 28-year-old woman has had diabetes mellitus since she was an adolescent. She is 8
weeks pregnant and is admitted under the care of Nurse Charm. Hyperglycemia during her
first trimester will have what effect on the fetus?
*

1/1

a. Hyperinsulinemia.
b. Excessive fetal size.
c. Malformed organs.

d. Abnormal positioning.

19. Methylergonovine (Methergine) is prescribed for a client with postpartum hemorrhage.


Before Charm administers the medication, she contacts the health care provider who
prescribed the medication if which condition is documented in the client’s medical history?
*

0/1

a. Hypotension
b. Hypothyroidism
c. Diabetes mellitus

d. Peripheral vascular disease


Correct answer
d. Peripheral vascular disease

20. What resources should a nurse use in teaching her postpartum client? Select all that apply.
1. Pamphlets
2. Audiotapes
3. Nurse specialist
4. Videotapes
*

0/1

a. 1, 2, 3, 4

b. 1 and 3
c. 1, 2, 4
d. 2 and 4
Correct answer
c. 1, 2, 4

21. Dina is a pregnant mother living in Sta. Ana, Manila. She has given birth to a twin, one
child, and had an abortion. Which of the statements below CORRECTLY applies, given the
obstetrical history of Dina?
*

0/1

a. Since there has been a twin, Jolina is G4P4


b. Since there was an abortion, Jolina is G4P3
c. Jolina is Gravida 3 Para 2 (G3P2)

d. Jolina is Gravida 4 Para 2 (G4P2)


Correct answer
d. Jolina is Gravida 4 Para 2 (G4P2)

Situation
The medical nurse has been assigned as a floater nurse in the pediatric hematology ward for a night
shift.

22. Which of the following would the nurse identify as the priority nursing diagnosis during
a toddler’s vaso-occlusive sickle cell crisis?
*

1/1

a. Ineffective coping related to the presence of a life-threatening disease


b. Decreased cardiac output related to abnormal hemoglobin formation
c. Pain related to tissue anoxia

d. Excess fluid volume related to infection

23. Which statements by the mother of a toddler would lead the nurse to suspect that the child
has iron-deficiency anemia? Select all that apply.
1. “He drinks over 3 cups of milk per day.”
2. “I can’t keep enough apple juice in the house; he must drink over 10 ounces per day.”
3. “He refuses to eat more than 2 different kinds of vegetables.”
4. “He doesn’t like meat, but he will eat small amounts of it.”
5. “He sleeps 12 hours every night and take a 2-hour nap.”
*

0/1

a. 1
b. 2
c. 1 and 2
d. 3 and 4

Correct answer
c. 1 and 2

Situation
A nurse is caring for several patients in the LR/DR complex. The following questions apply:

24. What is the primary presenting symptom in a client with abruption placentae?
*

0/1

a. Concealed hemorrhage
b. Oliguria
c. Abdominal pain
d. Rupture of membranes

Correct answer
c. Abdominal pain

25. A pregnant mother, 32 weeks gestation, was admitted to the OB ward for significant
bleeding in different mucosal areas. The physician found out that she has dead fetus
syndrome. What could be the reason for the bleeding?
*

0/1

a. HELLP syndrome
b. Placenta previa

c. Disseminated intravascular coagulation (DIC)


d. Idiopathic thrombocytopenic purpura (ITP)
Correct answer
c. Disseminated intravascular coagulation (DIC)

26. The nurse in the labor room monitors a patient in her transitional phase for interval of
contractions. She measures this:
*
1/1

a. From the beginning of one to the beginning of the next contraction.


b. From the start of a contraction to the end of the same contraction.
c. From the end of a contraction to the beginning of the next contraction.

d. From the beginning of one contraction to the end of the next contraction.

27. In the labor room, a pregnant patient was being assessed for cervical dilation and uterine
contraction. Which of the following findings tells the nurse that the patient is on the active
phase?
*

1/1

a. 3cm dilated, contractions every 15-30 mins


b. 7cm dilated, contractions every 3-5 mins

c. 8cm dilated, contractions every 3-5 mins


d. 10cm dilated, contractions every 2-3 mins

28. When PROM occurs, which of the following provides evidence of the nurse's
understanding of the client's immediate needs?
*

1/1

a. The chorion and amnion ruptures 4 hours before the onset of the labor.
b. PROM removes the fetus most effective defense against infection.

c. Nursing care is based on fetal viability and gestational age.


d. PROM is associated with malpresentation and possible incompetent cervix.

29. Dina is now in her second stage of labor; Cervix 8cm. Nurse R puts her in lithotomy
position. One of the dangers of this position that Nurse R should watch for in Dina is
___________.
*

0/1
a. Hyperventilation
b. Hypertension

c. Hypotension
d. Tachypnea
Correct answer
c. Hypotension

30. If Dina has a long second stage of labor and needs to be placed in Lithotomy position for
a long time, Nurse R should implement the following safety measures, EXCEPT
___________.
*

1/1

a. Wrap legs with elastic bandages

b. Put 2 pillows under her head


c. Place a small pad under her sacral area
d. Put a rolled towel under Dina’s right hip

31. Nurse R understands that Dina needs more support and encouragement during the PEAK
and MOST painful phase of uterine contractions, which is called ____________.
*

0/1

a. Decrescendo
b. Decrement
c. Increment

d. Acme
Correct answer
d. Acme

32. Before the INITIAL vaginal examination commonly referred to as internal examination,
what must Nurse R check FIRST?
*
0/1

a. Blood pressure for hypotension or hypertension


b. Heart rate of Dina for chest compression.
c. Abdominal palpitation for signs of fetal distress.
d. Abdominal palpitation for fetal lie and position, and fetal heart tone

Correct answer
c. Abdominal palpitation for signs of fetal distress.

Situation
Nurse Charie is assigned to several patients in the orthopedic ward.

33. A computed tomography (CT) scan is one of the diagnostic evaluations used for
musculo-skeletal function. Which of the following is NOT an indication of the procedure?
*

0/1

a. To measure the girth of an extremity.


b. To visualize and assess tumors.

c. To identify the location and extent of fractures in areas that are difficult to evaluate.
d. To assess severe trauma to the chest.
Correct answer
a. To measure the girth of an extremity.

34. Which of the following conditions is NOT a factor that will aggravate or increase the
blood pressure of a patient?
*

0/1

a. Having post-operative pain


b. Brisk walking in a treadmill

c. Suffering from acute renal colic


d. Asthmatic attack
Correct answer
a. Having post-operative pain

35. Another patient of Nurse C is Mrs. D, a 45-year old store owner, with a beginning
thrombophlebitis. Which of the following instructions is BEST to be given to a client with
this condition?
*

0/1

a. Massaging her legs when tired


b. Pillows be put behind the knees when lying on bed

c. Wearing support stockings one size larger


d. Apply heat in the affected area
Correct answer
d. Apply heat in the affected area

36. In the assessment of peroneal nerve function in relation to movement, instruct the patient
to _______.
*

1/1

a. Dorsiflex the foot and extend the toes

b. Prick the skin midway between the thumb and second finger
c. Plantar flex toes and foot
d. Prick the skin midway between the great and second toe

Situation
Nurse Clotilde is a nursing supervisor in the medical ward. Having the adequate knowledge in nursing
management, the following questions apply:

37. Nurse X knows that one of her staff is experiencing burnout. Which of the following is
the BEST thing for to do?
*

1/1

a. Remind her to show loyalty to the institutions.


b. Ignore her observations; it will be resolved even without intervention.
c. Let her staff ventilate her feelings and ask how she can be of help.

d. Advise her staff to go on vacation.

38. She knows that the performance appraisal consists of the following activities, EXCEPT
___________.
*

1/1

a. Setting specific standards and activities for individual performance.


b. Using agency standards as a guide
c. Focusing activity on the correction of identified behavior

d. Determine areas of strengths and weaknesses

Situation
Mr Bagumbaran came to the ER because of sharp troubling pain. After his surgery, he claimed pain is
felt even he is asleep.

39. In order for the nurse to recall the location of pain, he has to_______________.
*

1/1

a. Asks for onset and duration


b. Mark the painful area in a body diagram

c. Asks for facial expression


d. Asks verbal description using pain intensity scale

40. When a client is on prolonged pain therapy, the nurse should watch for____________.
*

0/1

a. Tolerance to drug
b. Allergic reaction to drug
c. Drug resistance
d. Addiction to drug

Correct answer
a. Tolerance to drug

Situation
Mariane Joy, a 42-year old teacher with cardiac ailment, nervously informs the doctor that her goiter
is getting bigger and distracts her while swallowing food. The physician who examined her instructed
the nurse to admit Myreen and to prepare her for surgery after medical clearance.

41. Based on your knowledge, Patient Mariane Joy, who has a history of cardiac illness,
should not be given an enema before surgery. Which of the following reasons inhibits the
order of enema for Patient Mariane Joy? Enema ____________:
*

1/1

a. Paralyzes the peristalsis movement and increases abdominal pain– enema increases peristalsis by
distending the bowel or irritating the intestinal mucosa
b. Produces vagal stimulation that is dangerous to cardiac patient

c. Causes constipation and fecal impaction after the surgery – on the contrary, enemas relieve
constipation
d. Enema results to increased water absorption in the bowels

Situation
Mc Roldan, 16-year old, a foreigner was admitted in the medical ward due to abdominal pain, nausea
and vomiting by Nurse Eclarin.

42. In initiating care for patient Mc Roldan, which of the following would be an
APPROPRIATE question to be asked by Nurse Tessie in her assessment?
*

1/1

a. Since this is doctor’s order, you have to drink ice water, instead of hot tea.
b. Do you have any books I could read about people of your culture?
c. Do you need to set aside your cultural practices, and comply with hospital rules and regulations?
d. Is there anything I am doing that is not acceptable to your culture?
43. Nurse Eclarin respects cultural practices integration in her nursing care plan. Which of
the following nursing action is MOST represented of the culturally competent nurse?
*

0/1

a. Help patient Kenneth to learn and understand the language


b. Explain and validate health knowledge and beliefs of Patient Kenneth with that of the hospital

c. Help Patient Kenneth identify ways to relate more to the culture where they now resides
d. Ask patient Kenneth to help Nurse Tessie in knowing more the culture of his origin
Correct answer
d. Ask patient Kenneth to help Nurse Tessie in knowing more the culture of his origin

44. Patient Mc Roldan’s family requests time for spiritual healing process in the hospital.
This is allowed by Nurse Eclarin and hospital because it______________________.
*

1/1

a. Gives fulfillment and meaning to the patient and family

b. Demonstrate people being responsible for their life patterns


c. Is non-denominated community service
d. Formalizes a religious dogma

45. Positive Practice Environment (PPE) influences healing process. Which of the following
ways can help Nurse Eclarin create a healing environment?
*

0/1

a. Ensure that relatives and friends visit the patient

b. Empower clients to make healthy decisions for themselves


c. Place television in each room of the hospital
d. Ensure that staff nurses does not experience burnout
Correct answer
d. Ensure that staff nurses does not experience burnout
Situation
Tisoy, 28 years old, has been diagnosed with Diabetes Mellitus. She was advised by her family
physician to be admitted to undergo preservation for insulin therapy. Her blood sugar ranges from 200
to 210 mg/dL. At 6 am, Nurse Gellie administered her insulin injection. After 2 hours, the patient
complained of cold clammy perspiration, chilly sensation and abdominal discomfort.

46. Which of the following PRIORITY nursing actions should the nurse perform?
*

1/1

a. Give her biscuit to eat

b. Do urine testing for sugar


c. Provide her warm blanket
d. Take blood pressure and put her on bed rest

47. Patient Tisoy has been classified to have a type II Diabetes Mellitus. Which of the
following is NOT a typical manifestation of individuals with this condition?
*

1/1

a. Frequency of urination
b. Increased craving for food
c. Increased thirst
d. Weight loss

Situation
Mr Franky, 58 years old is admitted to the pay ward because of respiratory problem. The nurse
initiated oxygen treatment by mask but the client refuses despite the encouragement by the wife. The
client is aware of the benefits of the treatment.

48. Which of the following should be given priority?


*

1/1

a. Ask the opinion of the wife


b. Conduct consensus building
c. Let the attending physician decide on the necessity of the treatment
d. Respect the decision of the client

49. You are taking care of Mr. Mr Franky who is on the last cycle of radiation therapy for his
lung cancer. You should instruct Mr. Franky to
*

0/1

a. Brush teeth and gums vigorously after meals


b. Wait one hour after treatment before eating

c. Use mouthwash containing alcohol every 2 hours


d. Avoid drinking hot fluids
Correct answer
d. Avoid drinking hot fluids

Situation
A nurse is caring for several patients in a pediatric unit. The following questions apply:

50. What is the appropriate response to a mother who is anxious to have her child undergo
herniotomy?
*

1/1

a. You cannot do anything about it.


b. The case is an emergency. If he does not undergo surgery he will die.
c. What can are your concerns about the procedure?

d. Why do you feel that way? Please tell me.

51. At 4 months, what health teaching must be communicated to parents to promote optimum
safety?
*

0/1

a. Raise side rails high.


b. Remove small objects from floor.
c. Remove poisonous substances from low areas.
d. Provide head gear for temper tantrums.
Correct answer
c. Remove poisonous substances from low areas.

52. A 3-year old client is prescribed with digoxin therapy. His resting heart rate is 60 bpm
and has vomited twice. After taking prescribed doses of digoxin, which of the following
would show toxicity?
*

0/1

a. Bradycardia
b. Vomiting

c. Lethargy
d. Increased appetite
Correct answer
c. Lethargy

53. The parents are planning to give their child ginger candy for cough. How will the nurse
respond best?
*

0/1

a. Let him have some and let’s see what will happen.
b. I will get a prescription for that.
c. It is not prescribed by the physician.
d. It is better to give child natural ginger rather than the candy form

Correct answer
a. Let him have some and let’s see what will happen.

54. A child is admitted to the hospital with a diagnosis of Wilms tumor, stage II. Which of
the following statements most accurately describes this stage?
*
0/1

a. The tumor is less than 3 cm. in size and requires no chemotherapy.


b. The tumor did not extend beyond the kidney and was completely resected.

c. The tumor extended beyond the kidney but was completely resected.
d. The tumor has spread into the abdominal cavity and cannot be resected.
Correct answer
c. The tumor extended beyond the kidney but was completely resected.

55. An 11-year-old boy is admitted to the pediatric unit in traction with a fractured femur
sustained in a motorcycle accident. His uncle, who was driving the cycle when the accident
occurred, received only minor injuries. The child tells the nurse that his uncle was not to
blame for the accident. He is “the best motorcycle rider in the
world.” The nurse interprets this to mean that the child is exhibiting which defense
mechanism?
*

1/1

a. Denial
b. Repression
c. Hero worship

d. Fantasy

56. The most common technique of male masturbation involves:


*

1/1

a. Grasping and stroking the shaft of the penis.

b. Inserting the penis into objects.


c. Rubbing the legs together.
d. Running a stream of warm water over the genital region.

Situation
Conducting Research is one of the major roles of the nurses both in hospital and community settings.
To be able to develop such competencies, the nurse has to undergo an actual conduct of the research
process.

57. Which of the following statements BEST described a researchable problem?


*

0/1

a. Responses of parents toward having children with congenital heart diseases.


b. What is the relationship between relaxation technique and relief of pain of post CABG patients in
the surgical coronary care unit.
c. Incidence of medication errors and reporting practices of Health Care Professional in a teaching
hospital
d. To what extent do pre-operative teaching affect the length of hospitalization of patients going for
surgery

Correct answer
b. What is the relationship between relaxation technique and relief of pain of post CABG patients in
the surgical coronary care unit.

58. Weight is taken as a baseline measurement of obese female adolescents as study subjects
for a weight reduction program. This is repeated to note any changes. This pre-test is done
to___________________.
*

0/1

a. Determine whether the instrument is defective


b. Assess if research design is appropriate to the problem identified

c. Evaluate whether the instrument is defective


d. Obtain preliminary data before a treatment is conducted by the researcher
Correct answer
d. Obtain preliminary data before a treatment is conducted by the researcher

59. A Nurse researcher is using ACCU-CHEK a monitoring kit to test presence of Diabetes
Mellitus among her study subject. How do you classify this type of measurement?
*

0/1
a. Microbial

b. Cytological
c. Physiological
d. Chemical
Correct answer
c. Physiological

60. Nurse Joan, wanted to conduct a study using quasi-experimental design. This design will
need a __________.
*

1/1

a. Retrospective evaluation
b. Field setting for the study
c. Comparable group
d. Manipulation of the independent variable

Situation
Mr See is being prepared for a major surgery. Legal preparation for surgery consists of checking all
the required forms for the operation. Equally important is to make sure that the patient is physically,
psychologically, and emotionally ready for the procedure.

61. Informed consent is a process that gives the patient opportunity involved in his or her
care. As patient advocate, the nurse ensures the following three conditions are present to
make consent valid, EXCEPT:
*

1/1

a. Adequate disclosure of the diagnosis by the physician


b. Comprehension of information by the patient before the operation
c. Patient voluntarily giving consent
d. Forms signed by any close relative or watcher
62. The patient asks you, “What do you think of my surgeon?” You answered “hmmmmm…
he is not really the best one and he seems not to care for patient...” As a result, the patient
switches to another surgeon. The latter may have grounds to sue you for _____________.
*

1/1

a. Slander

b. Invasion of privacy
c. Malpractice
d. Libel

63. One of your patient’s visitors whisper to you, “I hope you will not try to revive my dear
friend if her heart stops as she has already suffered a lot.” The correct response is
___________.
*

1/1

a. “That decision is up to the physician”


b. “We are all trained in cardiopulmonary resuscitation”
c. “There is a ‘ Do not resuscitate’ order in her chart”
d. “ I understand your concern, but I can’t discuss this matter with you”

63. According to the Joint Commission, the most frequently cited factor in sentinel
(unanticipated) events that leads to a patient’s serious physical or psychological injury is
______________________.
*

0/1

a. Confusion within the health team


b. Miscommunication among health team members

c. Incompetence by a team member


d. Policy changes are not followed by adequate and consistent staff education
Correct answer
c. Incompetence by a team member
Situation
Nurse Honelette is employed in hospital “X” and assigned in the Medical Ward for a year now. The
nurse supervisor ordered her to proceed immediately to the Surgical Ward as a reliever to another
nurse who went on emergency sick leave. She was not oriented in the Surgical Ward and the unit was
very busy.

65. Which of the following is the MOST appropriate action of Nurse Honelette
*

1/1

a. Request the nurse supervisor to assign a more experienced nurse reliever


b. Refuse the order of the nurse supervisor and stay put in the medical ward
c. Comply with the order of the nurse supervisor
d. Request the nurse supervisor to give her brief orientation before compliance

66. Nurse Honelette has an expired license but promises to renew her license in due time.
Which of the following violation can she be charged if she participated in home health care
activity?
*

0/1

a. Misdemeanor
b. Grave coercion
c. Felony
d. Negligence

Correct answer
a. Misdemeanor

Situation
Health Education is an area of nursing practice when the nurse can be creative and independent in the
work setting. The following questions apply.

67. A nurse is developing a Teaching plan for Patricia 18 year old with Bronchial Asthma.
She has an order for discharge. Which part of the teaching plan should be given PRIORITY?
*
0/1

a. Quick relief medicines as ordered

b. Avoid contact with fur-bearing pets


c. Avoid going to malls
d. Wash bed sheets in warm water
Correct answer
b. Avoid contact with fur-bearing pets

68. Nurse Karen is teaching Michel, an asthmatic, on how to use the Spirometer. She should
instruct the client to have the mouthpiece________________.
*

0/1

a. Place into the mouth and have regular breathing


b. Place into the mouth and have a fast deep breath
c. Place into the mouth and inhale slowly
d. Place into the mouth and exhale slowly

Correct answer
c. Place into the mouth and inhale slowly

69. Nurse Kim is teaching a client on how to use metered dose inhaler to prevent asthmatic
attack while in the hospital. She should instruct the client to do the following
__________________ EXCEPT.
*

1/1

a. Keep the head of the bed at 15 degree angle

b. Do oral care after use of the inhaler


c. Use the inhaler before she take her meals
d. Use the inhaler as ordered

Situation
The TQM nurse reported to their director that there are a lot of medication errors committed by nurses
and doctors in the hospital for the past 3 months. They made a decision to conduct a review of all the
cases with these errors. A clinical enhancement on drug administration was strongly recommended by
going back to the basics. The following questions apply:

70. In order to prevent error in insulin injection which of the following safety measure should
you do?
*

1/1

a. Leave the drug in the client refrigerator and ask it when needed.
b. Double check with your head nurse calculations and consider high “alert” drugs.

c. Compute the drug, proceed to the patient, and administer the drug.
d. Request the dietary department to keep the drug until needed.

Situation
Jayson is a 72 year old who is a smoker and a social drinker. He consulted the OPD because of rectal
bleeding. His tentative diagnosis is colorectal cancer. He was advised by the doctor for admission.

71. Which of the following would you likely expect as a specific complaint of the client
during your initial health history taking?
*

0/1

a. Projectile vomiting
b. Bouts of hematemesis

c. Change in bowel habits


d. Passing out white watery stools
Correct answer
c. Change in bowel habits

72. Colonoscopy has been ordered. Which of the following is not advisable for the client to
do?
*

0/1

a. Has to drink the electrolyte laxatives day before the procedure


b. Can take PRN medications when taking electrolyte solution
c. Can have liquid diet before the procedure.
d. Chilled electrolyte solution is allowed.

Correct answer
c. Can have liquid diet before the procedure.

Situation
Ms. Marjorie, 42 years old, has been manifesting thyroid enlargement and bulging of the eyeballs. She
has been complaining of dryness of the cornea even it is not a hot a season. She was advised by a
friend-doctor who is practicing in a nearby hospital to be admitted for further work-up.

73. You plan to conduct physical examination of the enlarged thyroids gland. What is the
MOST APPROPRIATE procedure to be done to examine Ms. Marjorie?
*

1/1

a. Have her drink cold water and measure the size of the thyroid gland
b. Request Ms Janice to sit on the chair and let her hyperextend the neck during palpation

c. Inspect and percuss the thyroid gland for dullness


d. Conduct a deep palpation to evaluate thickness of the enlarged thyroid gland

74. You have formulated a nursing diagnosis, "Risk for Corneal Injury R/T Exophthalmos"
for patient Marjorie. Which of the following interventions in the MOST appropriate to
prevent this problem to happen?
*

1/1

a. Apply artificial tears if ordered

b. Have an intraocular exercise as tolerated


c. Put patient Janice in upright position
d. Include salt restriction in her diet

Situation
Ashryl, 35 years old, consulted the OPD because of pain and abdominal distention. She also
complained of frequent urination in small amount. Doctor's impression is presence of kidney stone.
75. The client is asked to describe the pain she is experiencing. Which of the following
should the Nurse expect?

1. Intense deep ache in the costo-vertebral region


2. Pain usually radiates anteriorly and towards the bladder
3. Wavelike pain that radiates down to the thigh and genitalia
4. Acute excruciating pain, colicky in nature that radiates to genitalia
*

0/1

a. 1 and 2

b. 1 only
c. 2 and 3
d. 1,2,3 and 4
Correct answer
d. 1,2,3 and 4

76. Lithotripsy was ordered for Ashryl. She asked the Nurse "What is this procedure all
about? The nurse replied, " It __:
*

1/1

a. Will break up the stones in the calyces of the kidney"


b. Is a procedure to extract renal stones"
c. Will create electrical discharge to break the stones in the kidney"

d. Will visualize the stone and destroy it"

77. Ashryl is preparing to go home. Which of the following is NOT an essential component
of your health instruction?
*

1/1

a. Engage in outside activity so she can sweat a lot

b. Drink 8-10 glasses per day


c. Restrict intake of protein to 60 g/day
d. Take non steroid drugs for pain

Situation
Mr. Kerro, a 62-year old male, complains of shortness of breath, dyspnea on exertion, palpitation, and
expectorating frothy, blood-tinged sputum. He was brought to the emergency room of Hospital C.
During the interview, he also complained of easy fatigability and weakness.

78. Mr. Kerro has been diagnosed with heart failure. Nurse Joncel expects this patient to not
exhibit the following signs and symptoms EXCEPT:
*

0/1

a. Pulse rate of 48 bpm


b. Murmurs
c. Respiratory rate of 18 cpm

d. Strong, bounding pulse


Correct answer
b. Murmurs

79. Nurse Joncel prepares to perform the following interventions for the immediate
management of Mr. Kerro excluding:
*

0/1

a. Establish an IV line and administer PNSS immediately


b. Monitor heart rate and dysrhythmia by using a cardiac monitor
c. Continuously assess level of consciousness
d. Provide reassurance and support to the patient

Correct answer
a. Establish an IV line and administer PNSS immediately

Situation
Nurse Ayanah was invited to be a member of the ethics committee of Hospital D, a training hospital.
A prerequisite to her formal induction into the committee was for her to complete a research.

80. Before and during the conduct of her research, she is guided by the following ethical
principles excluding:
1. Protect the privacy of the subjects
2. Get the informed consent of the subjects
3. Avoid injury or harm to the subjects
4. Treat the subjects fairly and justly
*

0/1

a. 1 & 2
b. None of the options
c. All but 3
d. All of the options

Correct answer
b. None of the options

81. Another important ethical principle that guides nurse researchers is the principle of
justice. Nurse Ayanah knows that this principle connotes:
*

0/1

a. Fairness and confidentiality


b. Fairness and equality

c. Fairness and equity


d. Fairness and anonymity
Correct answer
c. Fairness and equity

82. Apart from the aforementioned ethical guidelines and principles, Nurse Ayanah is aware
that which of the following codes of conduct has endured through the ages?
*

0/1

a. Code of Ethics for Professionals

b. Phairpine Constitution
c. Honor Code
d. Ten Commandments
Correct answer
d. Ten Commandments

83. Getting the informed consent of research participants show that the nurse honors the
principle of autonomy. However, there are instances where participants become incapable of
giving informed consent. These groups of participants exclude which of the following:
*

0/1

a. Jestoni, a 19 year-old male who is paraplegic


b. Karlyn, a 17 year-old college student
c. Ayesha, a 34 year-old mother who is diagnosed with Schizophrenia with active positive symptoms
d. All of the options

Correct answer
a. Jestoni, a 19 year-old male who is paraplegic

Situation
As a staff nurse in a government hospital, you have been exposed to varied cases of clients with
endocrine problems. Your nursing responsibility starts from admission to discharge which is a domain
of your competencies.

84. When the sympathetic nervous system is stimulated in the case of pheochromocytoma,
you expect which of the following signs?
i. Hypertension
ii. Headache
iii. Hyperhidrosis
iv. Hypermetabolism
*

1/1

a. 3 and 4
b. 1 only
c. 1, 2, 3, and 4

d. 2 and 3

85. Which of the following drugs can induce hypertensive crisis in Pheochromocytoma?
*

0/1

a. Tricyclic antidepressant
b. Corticosteroid
c. Respiratory stimulant

d. Radio iodine therapy


Correct answer
a. Tricyclic antidepressant

86. In the presence of pheochromocytoma, the diagnostic test which is expected to be


elevated is ___________.
*

0/1

a. Serum thyroid hormone levels


b. Albumin globulin test
c. Urine cyclic adenosine mono phosphate

d. 24 hours urine collection for vanillylmandelic acid (VMA)


Correct answer
d. 24 hours urine collection for vanillylmandelic acid (VMA)

Situation
Janine is the Nurse on duty in the medical ward and many of her patients are suffering from problems
of oxygenation.

87. The following are relevant data to be documented when taking the health history of a
client with anemia EXCEPT:
*

1/1

a. Alcohol intake
b. Fatigue and weakness
c. Dietary intake
d. Episodes of bleeding
88. A client with congenital heart disease is suffering from thickening of the skin under his
fingers due to chronic hemoglobin desaturation. Which of the following specific term should
Ime use to accurately describe MOST the client’s condition in the chart?
*

1/1

a. Peripheral cyanosis
b. Pallor of the finger tips
c. Peripheral neuropathy
d. Clubbing of the fingers

89. Mr. Gabby is with left sided heart failure. Janine’s documentation of her assessment
findings will include the following, EXCEPT______________.
*

0/1

a. Dependent edema
b. Pulmonary crackles
c. Difficulty of breathing
d. Cough

Correct answer
a. Dependent edema

90. A client is on a diuretic therapy. Expected entry in patient’s chart should include the
following information, EXCEPT:
*

1/1

a. Serum electrolytes monitored


b. Intake and output recorded
c. Lasix administered at 8 o’clock in the evening
d. Weight is taken before drug is given

Situation
The Registered Nurses in the practice of the profession are expected to demonstrate an upright and
ethical values inside and outside of their work settings.

91. The primary purpose of the Nurses’ Code of Ethics includes which of the following?
*

1/1

a. Provide guidelines for the care provided to patients.


b. Establish right attitude and behavior of nurses.
c. Serves as a guide in making morally upright decision related to nursing practice.

d. Regulate the practice of the profession

92. The ethical principle which considers the persons unconditional worth in determining his
own destiny is ______
*

1/1

a. Self-determination
b. Fidelity
c. Autonomy

d. Justice

93. Nurses can improve ethical decisions by:


*

0/1

a. compliant to the Code of Ethics


b. attending conferences and symposis on Ethics

c. compliant to policies and laws


d. being a member of the ethics committee
Correct answer
a. compliant to the Code of Ethics

94. What ethical principle is applied in clinical situation, when respirator is available for two
patients in shock?
*

0/1

a. stewardship

b. inviolability of life
c. justice
d. respect for person's dignity
Correct answer
c. justice

Situation
Miss Ahya and Mister Aaron are province mates and graduated from the same nursing school. After
two years of staff nursing experience they decided to go home to their province. Nurse Ahya was
employed as a school nurse while Mr. Aaron joined a tertiary health facility an infection control
nurse. One day, a group of school children approached Nurse Ahya because of fever, cough and
fatigue.

95. What PRIORITY nursing actions should Nurse Ahya implement?


*

0/1

a. Give cough syrup and antipyretic in the children

b. Teach the children to cover their mouth when coughing


c. Call parents and teach them hand washing technique
d. Check if the children have been immunized with BCG
Correct answer
b. Teach the children to cover their mouth when coughing

96. One of the children was complaining of headache and epistaxis. Which of the following
nursing measures should Nurse Ahya undertake first?
*

1/1
a. Put client on bed in supine position
b. Have the child sit upright with head tilted forward

c. Bring the child to the nearest hospital


d. Pinch the soft outer portion of the nose for 6 to 15 minutes

97. Nurse Ahya conducted a visit to the community where the children live. She found out
that the brother of one of the school children was just newly discharged from the hospital
because of Hepatitis A. Which of the following precautionary measures should she give
HIGHEST PRIORITY to the family members?
*

0/1

a. Dispose properly syringe used by patient in covered containers


b. Avoid droplet infection

c. Use mask when caring for the patient


d. Wash hands before and after handling excretory and secretory materials of the patient
Correct answer
d. Wash hands before and after handling excretory and secretory materials of the patient

98. Nurse Aaron is working with the infection control area and participated admitting
patients affected with suspected Viral Infection. Which of the following action proved to be
effective to prevent incidence of cross contamination in the work setting?
*

0/1

a. Health care professionals should observe standard precautions


b. Come up with a policy providing immunization to the health care
c. Provide dispenser for alcohol-based hand wash in and outside profession of patients room

d. Post EBOLA health precautions in all areas of the hospital


Correct answer
a. Health care professionals should observe standard precautions
99. A 30-year old G6P5 woman at 12 weeks has just begun prenatal care. Her initial
laboratory reveals that she has human immunodeficiency virus (HIV) infection. What would
be a priority evidence-based nurse information for this patient?
*

0/1

a. Breastfeeding is not recommended due to the great risks to the infant

b. Pregnancy is known to accelerate the course of HIV disease in the mother


c. Medication for HIV infection is safe and can greatly reduce transmission of HIV to the infant
d. Breastfeeding will potentiate the transmission of HIV from the mother to the child
Correct answer
c. Medication for HIV infection is safe and can greatly reduce transmission of HIV to the infant

100. There is a senior student where Nurse Aaron is assigned. Which of the following
infection control activity is APPROPRIATE to be delegated to the senior students?
*

1/1

a. Assessment of patients with chronic respiratory problems


b. Take care of patient with pneumonia in acute phase of his illness
c. Demonstration of hand washing to patients and relatives.

d. Disinfection of BP cuff, thermometer while patients are in the hospital

RECALLS 3 - NP3
Total points49/100

Situation
Mr. Jayson, a 42-year old engineer, was brought to the emergency department of Hospital C. The
nurse accommodating him learned that he complains of sudden, severe headache, numbness of the
right side of his body, and difficulty speaking. The nurse immediately administered oxygen to the
client.

1. Through this intervention, the risk for the following complication is lessened including:
*

1/1
a. Fluid accumulation in the lungs
b. Pulmonary emboli
c. Rebleeding
d. Increased intracranial pressure

2. The attending physician orders that the client undergo a non-contrast CT scan. The nurse
understands that this diagnostic examination is performed to determine:
*

0/1

a. If the event is ischemic or hemorrhagic


b. The extent of damage to Mr. Cadiz's motor functioning

c. The appropriate treatment for Mr. Cadiz


d. The size of his brain
Correct answer
a. If the event is ischemic or hemorrhagic

3. Oxygen is immediately administered to a patient suspected to have had stroke in order


to:
*

0/1

a. Prevent hypoxia and hypercapnia


b. Prevent further neurologic deficits
c. Prevent respiratory deficit
d. Improve the respiratory function

Correct answer
a. Prevent hypoxia and hypercapnia

Situation
Nurse Binwag, a newly promoted senior nurse in Obstetrics ward (OB) is attending a seminar on
management and leadership in preparation for her work.
4. Nurse Binwag learns the five principles of goal setting in which the senior nurse must
provide enough time for OB nurse to improve performance. This is called__________
*

1/1

a. Challenge
b. Commitment
c. Feedback
d. task complexity

5. The nurse also learns that continuous training is a personal as well as an organizational
goal. Choose the statements that are true regarding continuous training
1. Training employees is an excellent investment and a cost to an institution
2. Continuous training is more of a personal responsibility than institutional
3. Cross training and job rotation provide on-going part-time learning experience
4. Select the best people when hiring employees and invest their retention through
continuous training
*

1/1

a. 3 and 4

b. 1 and 2
c. 1 and 4
d. 2 and 3

6. Noting the importance of Nurse-Patient-Relationship, Nurse Binwag reviewed


Hildegard Peplau’s Theory which identified three phases, the FIRST of which is when the
pregnant woman is________.
*

0/1

a. Feels the need to seek professional assistance


b. Demonstrates self-reliance in caring for herself
c. Understands the communication of Nurse Lovely regarding the services offered

d. Begins to have feeling belonging


Correct answer
a. Feels the need to seek professional assistance

Situation
You are an OB nurse in an out-patient department of a hospital. You encounter pregnant women with
complication

7. A 35-year old woman, on her 2nd trimester of pregnancy with insulin-dependent


diabetes mellitus, comes to you for some advice. What is the PRIORITY message for her at
this time?
*

1/1

a. Infants of diabetic mothers are big which can result in more difficult delivery
b. Breastfeeding is highly recommended and insulin use is not contraindicated
c. Achievement of optimal glycemic control is of utmost importance in preventing congenital
anomalies

Situation
Nurse Dory, a nursing staff applicant, passed both written and oral examinations. Because she knows
the head of office, she promised to submit all her credentials after she has “fix things up.” She was
appointed as Nurse I with a temporary status until she submits all her credentials, including her PRC
license. Her evaluation performance was satisfactory. After a year though, she had to renew her PRC
registration and identification (ID) card.

8. What action must the nursing administration do FIRST?


*

0/1

a. Report the matter to the head of office who had the discretion to appoint the nurse
b. Verify with the Professional Regulation Commission regarding the status of Nurse Dora
c. Confront Nurse Dora and terminate her

d. Write a letter to the Civil Service Commission for proper action to Nurse Dora
Correct answer
a. Report the matter to the head of office who had the discretion to appoint the nurse

9. It was found out that Nurse Dora did not pass the Nurse Licensure Examinations (NLE).
What legal action should be filled against her?
*
1/1

a. Dishonesty
b. Conduct unbecoming of professional
c. Malpractice
d. Misrepresentation

10. In case Dora have medication error during her tour of duty, the head of office can be
liable because of the law called ____________.
*

1/1

a. Unethical conduct
b. Respondeat superior

c. Politicking
d. Res Ipsa Loquitur

11. All Nurses must understand that after graduation they should pass the NLE. To be
registered in the roster, they should take the Professional Oath with a ________, EXCEPT.
*

1/1

a. Member of Sangguniang Panlalawigan


b. Governor of Philippine Nurse Association

c. Member of the Professional Regulatory Board of Nursing

12. A nurse is leading a community meeting at the partial-hospitalization program. One


group member talks constantly and interrupts the other patients. The most appropriate action
for the nurse to take is to
*

1/1

a. Explain to the member, after the meeting, that the group time should be shared.
b. Thank the group member for his contribution and ask the other members for their ideas
c. Ask the group members if they are satisfied with the way the group is working.
d. Remain silent and wait for another group member to speak up.

Situation
Laica, 45 years old, post-menopausal, consulted the physician in the OPD for abdominal pain. She has
been stressed, irritable lately. She is more relaxed when she smokes and drinks alcoholic beverages.
She was advised by the doctor to be admitted for a suspected duodenal ulcer.

13. Ms. Laica was ordered to have blood typing. The nurse is aware that the higher incidence
of duodenal ulcer belongs to what blood type?
*

1/1

a. A
b. B
c. O

d. AB

14. Gastrointestinal endoscopy was ordered in order to have better visualization of the
gastric mucosa. After the procedure, the patient developed high grade fever of 39.8 degrees
Centigrade with acute abdominal pain. What PRIORITY nursing intervention should the
nurse perform?
*

1/1

a. Assess the patient at once for possible perforation

b. Take vital signs every two hours


c. Give tepid sponge bath
d. Give relief medication as ordered

15. Which of the following conditions is contraindicated for nasogastric tube insertion and
need not be ordered by the physician? Patients ____:
*

0/1

a. With cranio-facial surgery


b. All of the options
c. With hypophysectomy
d. Recovering from gastric surgery
Correct answer
b. All of the options

16. Which of the following is an EXPECTED outcome in a patient with duodenal ulcer? The
patient will:
*

1/1

a. Join friends for social drinks every weekend


b. Engage self with smoking cessation program

c. No longer join sports program


d. Use Tylenol for control of pain

Situation
Jaina is a fourth year nursing student currently having her community immersion in Barangay Sto.
tomas. She, together with her professor, are is conducting a home visit with one of her patients, who is
diagnosed with an open-angle glaucoma.

17. In the middle of their conversation, the client raised a question to Jaina. She mentioned
that she has been prescribed with Latanoprost (Xalatan) by the municipal doctor. She asks
Jaina why she has to take this. Jaina responds that this medication:
*

1/1

a. Dilates the affected eye


b. Moistens the affected eye
c. Decreases the angle of glaucoma in her affected eye
d. Decreases the intraocular pressure in her affected eye

18. In utilizing the IMCI protocol, the nurse should initially?


*
0/1

a. Observe the condition of the child


b. Ask the mother if what is the problem of the child
c. Look for danger signs
d. Identify main symptoms

Correct answer
b. Ask the mother if what is the problem of the child

Situation
Arlene, 21 years old, is a law graduate. She wants to review for the Bar but thinks she is pregnant. She
said she has regular menses but does not know when ovulation usually occurs. This have something to
do her fertility period during her last sexual intercourse with her husband.

19. As a nurse, what would you tell Arlen regarding ovulation? The ovulation usually
corresponds to the life of the corpus luteum which occurs approximately _______________.
*

1/1

a. 14 days after the first day of the succeeding menstrual


b. 7 days after the first day of the succeeding menstrual
c. 7 days before the first day of the succeeding menstrual
d. 14 days before the first day of the succeeding menstrual

20. The nurse proceeded to take the menstrual history of Arlen to find out if she is likely to
be pregnant. Which of the following determines the date of onset of last menstrual period
(LPM)? It is the ___________.
*

0/1

a. Duration and character of the LMP

b. Implantation bleeding
c. Spotting after the LMP
d. Bleeding ceases before the last menstrual period (LMP)
Correct answer
d. Bleeding ceases before the last menstrual period (LMP)

21. The nurse also asked about Arlene’s secondary amenorrhea that would most likely
indicate her pregnancy. Secondary amenorrhea is cessation of menses for more than_______
months, after regular menstrual cycle has been established.
*

1/1

a. Five
b. Three
c. Six

d. four

Situation
The family of Roxas is fond of dogs. A vendor who entered the gate without notice is bitten by one of
the pet dogs named Bert. PHN Cords Attends to the vendor.

22. Which of the part of body of the vendor will be the MOST affected in terms of rabies?
It is the _________.
*

1/1

a. Buttocks
b. Head

c. Feet
d. hand

23. To protect the vendor from the dangers of rabies, PHN Cords advises him to clean the
wound thoroughly with soap and water, consult a physician and receive anti-rabies
vaccination. Which among the following vaccines can provide active immunity?
1. Purified vero cell vaccine
2. Human rabies immunoglobulin
3. Equine rabies immunoglobulin
4. Purified duck embryo vaccine
*

1/1

a. 1 and 4
b. 2 only
c. 3 and 4
d. 1 only

24. The vendor acquired rabies, what will PHN Cords do to protect those who took care of
him? He should administer____________.
*

1/1

a. Pre-exposure prophylactic treatment only for the family of Bert


b. Post-exposure prophylactic treatment only for the family of the vendor

c. Pre-exposure prophylactic treatment to Bert and the vendor’s families


d. Post-exposure prophylactic treatment to Bert and the vendor’s families

25. PHN Cord’s intervention to protect all residents who own pets, especially dogs,
should be done by_______.
*

1/1

a. Coordinating with city/ municipal agriculturist for immunization of all pets


b. Coordinating with city/municipal officials to make an ordinance on stray dogs
c. Massive campaign to families not to own pets at home
d. Massive campaign for responsible pet ownership

Situation
The Field Health Services and Information System (FHSIS) is recording and reporting system in
public health care in the Philippines

26. The following are the objectives of the FHSIS, EXCEPT:


*

1/1

a. Complete the picture of acute and chronic disease


b. Ensure data recorded are useful and accurate and disseminated in a timely, easy to use fashion
c. Minimize recording and reporting burden allowing more time for patient care and promotive
activities
d. Provides standardized facility-level data base which can be used for more in-depth studies

27 As a nurse, you should know the process of how these information are processed and
consolidated. The fundamental block of the FHSIS system is the ______________
*

1/1

a. Family treatment record

b. Output record
c. reporting forms
d. target/client list

28. The monthly field health service activity report is a form used in which of the
components of the FHSIS?
*

1/1

a. Target/client list
b. Output report
c. individual health record
d. tally report

29. In using the tally sheet, the recommended frequency in tallying activities and services
is_______.
*

1/1

a. Weekly
b. Quarterly

c. Monthly
30. To monitor clients client registered in long-term regimen such as the Multi drug
Therapy, which component of the reporting system will be most useful?
*

0/1

a. Output report

b. Tally report
c. target/client list
d. individual health record
Correct answer
c. target/client list

Situation
Nurse Navarrete takes care of the Ramos extended family who resides in the house owned by Nilda’s
mother, Marta. Nilda tends a variety store is married to Ramon, a government employee. They have
four children: Lester, 20 years old; Gina, 18 years old; Alex, 15 years old; and Celine, 12 years old.
Lester is a working student of his second year in computer technology course. Gina is a high school
graduate; Alex is in third year high school and Celine is in Grade six. There is one year old baby girl
who is a daughter of Gina. Gina, however, could not pinpoint the one who sired her child in as much
as she had multiple sex partners. This angered Ramon.

31. Though he recognizes the remorse of his daughter, Ramon still feels confused regarding
the situation. He said he tried his best to support his family and had always been considerate
and kind. He and his wife would always give them reminders and advice calmly and never in
a nagging manner. But still they failed as parents. The possible nursing diagnosis of Nurse
Oscar of his family is_____________.
*

1/1

a. Interrupted family process

b. Impaired parenting
c. Parental role conflict
d. Ineffective role performance

32. Nurse Navarrete ’s conversation with Gina revealed that the young woman still suffers
from a syndrome of failure: failure to complete one’s normal growth and development,
failure to complete education, failure to establish a vocation and become independent and
failure to have a life. The nurse Oscar’s intervention to this problem is_____________.
*

1/1

a. Linear approach with regards to the individual in the context of the family, community and culture
that will combat shame and guilt
b. Lay down the foundation of a future by trusting human association and developing mutual trust
initially with the nurse, then the family, and eventually the whole community
c. Focus on the factors that will help protect Gina towards proximal stimuli for healthy growth and
development to develop her resiliency in confronting current and future problem

d. Transform interactions among family members, strengthen specific roles and functions to
strengthen family system in order to eventually cope

33. In one home visit, Nurse Navarrete was approached by the 15 year old Reuben John. He
was asking about condom use. He said he has a girlfriend with whom he is madly in love
with but does not want her to get pregnant. Nurse Navarrete most practical and best advice
would be__________.
*

1/1

a. Postpone sex and suggest other ways to expressing love


b. Explain to him the difference between sex and love
c. Teach him by step-to-step correct, continuous and consistent condom use

d. Discourage him on having a girlfriend and focus more on his studies

Situation
PHN Ahya works in Barangay A and B in the Municipality of Mendez. One day, a neighbor summons
her to attend to a 7-year old boy with high grade fever.

34. Upon reaching the house, a local herbolaria, Nanay Feds was already attending to the
boy. She said that the boy played near the river and the bad spirits entered his body. The
LEAST appropriate remark the nurse make is _________________.
*

1/1

a. “Go on. Do what you have to do, then I will take over.”
b. “Nanay Feds, your intervention is entirely wrong.”
c. “It’s good you’re here. You can drive away the spirits that entered the boy’s body.”
d. “You have to be sure that all the evil spirits have been driven out of the boy’s body.”

35. After a few minutes, Nanay Feds took a big bowl of soup and gave to the boy. The boy
vomited but Nanay Feds continues to feed him. The BEST remark of the Nurse is
_____________.
*

1/1

a. “I also drink a soup when I get sick. How about you, Nanay Feds, do you do the same?”
b. “The soup could have been better if you put lemon grass on it.”
c. “You have to stop feeding him because he might aspirate the soup.”

d. “That’s correct. Increasing fluid intake will help lower down temperature.”

36. Finally, Nanay Feds took out from her pocket a dried rose flower and place it on the
boy’s forearm. She was about to cut the patient’s forearm. What would Nurse Ahya do
first?
*

0/1

a. Let it go
b. Tap Nanay Feds’s shoulder
c. Ask the herbolaria the rationale for the intervention
d. Explain the possible harm of the procedure

Correct answer
b. Tap Nanay Feds’s shoulder

Situation
The local health board established a reproductive health clinic in the main health center. Two nurses,
Hubert and Irene, were assigned to handle services to address problems related to sexuality,
reproductive health and fertility problems.

37. Nurse Valentin classifies cases according to the major categories of reproductive tract
infections. Which of the following is NOT part of such classification?
*

0/1
a. Iatrogenic infections as aftermath of invasive procedures like catheterization and intra-uterine
device (IUD) insertion

b. Urinary tract infections among male and female patients


c. Sexually-transmitted infections
d. Endogenous infections resulting from poor personal hygiene
Correct answer
b. Urinary tract infections among male and female patients

38. Irene handles the screening for gonorrhoea every two weeks among female sex workers
in the implementation of PD 856. In differential diagnosis of discharge among infected
clients, which of the following colors discharge will Irene take note to identify gonorrhoea
from other causes?
*

1/1

a. Greenish yellow as differentiated from mucoid white of trichomoniasis


b. Mucoid white as compared to grayish-white discharge of vaginosis
c. Grayish white as differentiated from mucoid white of chlamydia
d. Yellowish white as compared to trichomoniasis’ greenish-yellow

39. Nurse Valentin was invited by a women’s group to give a lecture on healthy sexuality.
In the expectation check, he noted that there are previous misconceptions expressed by the
participants. Which of the following statements are correct?
*

1/1

a. It is the obligation of the wife to give in to sex every time he asks for it
b. Sexuality is fluid and may change

c. Effeminate men are gays


d. Homosexuality, being gay or lesbian, is an abnormality

40. One of the clients was positive to Gonorrhea. Nurse Irene explained that gonorrhoea
and chlamydia, if left untreated can lead to Pelvic Inflammatory Disease (PID). Such
condition may cause infertility due to ______.
*

1/1

a. Foul smelling odor discharge which can kill the ovum


b. An unknown cause
c. Scarring which can lead to tubal occlusion

d. Purulent discharge which can kill the sperm

41. Nurse Hannah further explained that a test used to determine tubal patency using a
radiopaque material is the __________.
*

1/1

a. Post-coital infertility test


b. Sims Huhner test
c. Friedman’s test
d. Hysterosalpingography

Situation
Barangay Malanday is a barrio located on the far-east side of the municipality of Unanggapo. Kevin,
the community health nurse, developed an advocacy leaning towards mental health promotion.

42. Nurse Kevin understands that using the public health model, his patient is the:
*

1/1

a. Individual
b. Family
c. Community

d. Country

43. To grasp the community's overall mental health status - the incidence and prevalence of
mental illnesses – Kevin Feir employed epidemiological studies. In this situation, incidence
refers to the number of:
*
0/1

a. New cases of mental illness over a specified period of time in the population at risk
b. New cases of mental illness in the total population
c. People at risk to develop mental illnesses
d. All existing cases of mental illness over a specified period of time in the population

Correct answer
a. New cases of mental illness over a specified period of time in the population at risk

44. Nurse Kevin understands that primary prevention is a very important concept especially
in terms of his advocacy. He applies this concept through:
*

0/1

a. Program development and intervention for adolescents with mental illness

b. Reducing the rate at which new cases of mental illness develop


c. Early treatment of new cases of mental illnesses
d. Case-finding for the population at risk for mental disorder
Correct answer
b. Reducing the rate at which new cases of mental illness develop

45. Primary prevention entails mental health promotion. This activity can be considered
consistent with the principle of:
*

1/1

a. Autonomy
b. Interdisciplinary care
c. Multidisciplinary care
d. Self-help

Situation
Sexuality is a major issue in psychosocial health that is often not emphasized or considered taboo by
many nurses. Good thing you are a nurse who is nonjudgmental when it comes to issues in sexuality.
46. Joeven, 25, admits to you that he has multiple sexual partners. Which among the
following will you do regarding this?
*

1/1

a. Counsel him that it is shameful for men to have multiple sexual partners.
b. Educate him regarding safe sex practices.

c. Terminate care right away for it is dangerous to be dealing with people like him.
d. Refer him to a psychiatrist.

47. Marjorie comes to a clinic to complain of involuntary vaginal spasms that make it
difficult for her to have intercourse. This is referred to as:
*

1/1

a. Dyspareunia
b. Vaginismus

c. Vulvodynia
d. Vestibulitis

48. Which among the following medications can decrease sexual desire?
*

0/1

a. Phenytoin

b. Morphine
c. Atenolol
d. All of the above
Correct answer
d. All of the above
49. Homosexuality, heterosexuality, and bisexuality are varieties of which of the following:
*

1/1

a. Sexual orientation

b. Transgenderism
c. Gender identity
d. Intersex condition

Situation
Leadership and management are significant concepts for nurses. Being the head nurse of the medical
unit of Hospital A, Nurse Ivee applies various techniques of nursing management.

50. As Florence Nightingale theorized, nurses must utilize the environment of the patient to
assist him/her towards recovery. Nurse Ivee is aware that the biggest problem of hospitalized
clients is:
*

0/1

a. Ventilation

b. Hospital bed
c. Lighting
d. Noise
Correct answer
d. Noise

51. Nurses must ensure an infection-free environment for their clients. Ivee mentions to her
staff that the protocol that serves as the first line of defense against infection is:
*

1/1

a. Handwashing

b. Disinfection
c. Cleansing
d. Sterilization
52. Nurse Charlton Jao handles 16 patients during her shift.She delegated the routine
morning care of Patient X to Ashryl, a nursing aide. In this case, you are aware that the
person most accountable for the performance of care is:
*

0/1

a. Ashryl
b. Ivee
c. Charlton Jao
d. All of the above

Correct answer
c. Charlton Jao

53. Charlton Jao supervised Ahsryl during provision of morning care and advised him that it
is important that he understands how to provide care in order to:
*

1/1

a. Perform the procedure quickly


b. Explain the procedure to the client
c. Teach another nursing aide the procedure
d. Complete the procedure safely

54. As the week is about to end, Nurse Charlton Jao starts to make the daily case assignment
of her staff for the following week without getting their inputs. he often commands his
subordinates and strictly monitors their activities during their shift. Nurse Sherlyn, the chief
nurse, advises Charlton Jao to consider his staff, seek inputs from them, and to act as a
facilitator rather than a controller. This kind of approach is called:
*

0/1

a. Authoritarian
b. Bureaucratic
c. Laissez-faire
d. Democratic
Correct answer
d. Democratic

Situation
Nurse Angelica Lim is assigned as the charge nurse for the AM shift at the surgical unit. With this,
she knows that she is responsible to lead and manage the unit and the staff.

55. While doing her rounds, head nurse Angelica Lim observes that Yeng, a staff nurse,
performs wound care to her patient without observing aseptic techniques. Angelica Lim
knows that the best approach that she can use to assist the staff nurse is to:
*

1/1

a. Assign the staff nurse to a different unit


b. Ask another staff nurse to accompany and guide Inna when performing interventions
c. Discuss with the staff nurse her observation and collaborate to think of ways she can improve

d. Write a memo for Inna and mention that she will get fired if she does not improve her practice

56. Nurse Angelica Lim reviews the unit's record. She notices that Jestoni, another staff
nurse, has been frequently absent for the past 3 weeks, and this has affected the quality of
care received by their patients. Nurse Angelica Lim implements the most appropriate
strategy and:
*

1/1

a. Issue Jestoni a memorandum regarding his absence


b. Inform Jestoni that his absences can be a ground for termination
c. Talk to Jestoni about the concern and remind him of the standards of the hospital

d. Maintain an accurate record of attendance of the staff

57.Joncel is a new staff nurse in the surgical unit. He was assigned to care for Mr. Dexter
who is scheduled for craniotomy tomorrow. After administering the scheduled medications
for Mr. Dexter, Joncel knows that he is legally responsible to document on the patient's
chart which of the following:
*

1/1

a. Client's reaction to the drug

b. Client's civil status


c. Peak concentration of the drug
d. Therapeutic range of the drug

Situation
Nurse Xander is assigned as the medication nurse for the whole week. Because of overflow of
patients, their ward has recently catered to both adult and pediatric patients.

58. Xander is about to administer cotrimoxazole to a 6-year old pediatric patient. Which
among the following is least likely considered in calculating for pediatric dosages?
*

0/1

a. Weight
b. Height

c. Body surface area


d. Head circumference
Correct answer
d. Head circumference

59. Malou-Ang is a 9-year-old patient diagnosed with Gram- negative sepsis. She was
prescribed with meropenem IV. The usual adult dose of the drug is 500 mg IV q8h.How
much is the approximate dose per day for Vanessa if her body surface area (BSA) is 0.57 m2?
*

0/1

a. 168 mg

b. 503 mg
c. 240 mg
d. 720 mg
Correct answer
b. 503 mg

60 Xander Ford is given a patient with several co- morbidities. Which among the following
drugs has the longest duration of action?
*

1/1

a. Atorvastatin
b. Labetalol
c. Digoxin

d. Captopril

61. Regular insulin and NPH insulin are both ordered for a diabetic client. In mixing the two
insulins in a single syringe, which is an appropriate nursing action?
*

0/1

a. Shake vials in an up-down movement prior to withdrawing.


b. Excess NPH insulin is returned to its vial to reduce costs.

c. Withdrawing protaminated insulin first before the other.


d. None of the above
Correct answer
d. None of the above

62. The nurse administered Humulin R to a patient at 6am.At what time does the nurse
expect the patient to exhibit pallor, confusion, hunger, and abdominal pain?
*

0/1

a. 10 am – 1 pm
b. 7 am – 8 am
c. 8 am – 9 am
d. 6:30 am – 7 am
Correct answer
c. 8 am – 9 am

63. The nurse administered NPH to a patient at 12nn. At what time does the nurse expect the
client’s blood sugar to be relatively low?
*

0/1

a. 4 pm – 7 pm
b. 2 pm – 3 pm
c. 1 am – 4 am the next day

d. 1 pm – 2 pm
Correct answer
a. 4 pm – 7 pm

64. What is the appropriate gauge and length of a needle to be used in an obese adult for
insulin administration?
*

0/1

a. G25 and 3/8 inch

b. G24 and 5/8 inch


c. G25 and 1 ¼ inch
d. G24 and 1 inch
Correct answer
d. G24 and 1 inch

65. A client tells the nurse that the religious preference is Jehova’s Witness. Which is a true
statement regarding the use of blood products for this client?
*
1/1

a. Packs red cells are allowed in emergency and critical situations.


b. Blood cannot be used with heart-lung equipment.
c. The client may be given doses of erythropoietin (Epogen) to assist in the creation of new blood
cells.

d. Blood may be accepted if it is taken from a blood relative and used immediately.

Situation
Nurse Clotilde is a head nurse in a medical ward in Hospital X. The following questions apply:

66. You are a staff nurse working in a tertiary hospital. While charting, you see your charge
nurse documenting an administered medication even before it was given. What is your
priority nursing action?
*

1/1

a. Tell the charge nurse that doing so may affect her integrity and may harm the patient.

b. Report to the head nurse promptly.


c. You are under the charge nurse. You are not entitled to confront her.
d. Forget that you saw what the charge nurse did.

67. You observed a nurse logging into an account of another nurse in your ward. What is
your initial action?
*

0/1

a. Call the attention of the head nurse.

b. Report immediately to the nursing supervisor.


c. Approach the nurse and stop her from logging in.
d. Ignore the nurse. Every health care team member has the right to know the cases of patients.
Correct answer
c. Approach the nurse and stop her from logging in.
68. Which of the following is an example of intentional tort?
*

0/1

a. The nurse did not come to the patient’s room upon turning of the call light. Patient suffered a fall.
b. The patient reported he felt bleeding in his rectal area. The nurse did not respond, and later found
that the patient suffered hypovolemic shock.

c. Two nurses are talking about a patient in a cafeteria when another nurse heard the conversation.
d. A nurse administered 50mg of digoxin when it should be 25mg only.
Correct answer
c. Two nurses are talking about a patient in a cafeteria when another nurse heard the conversation.

69. A nurse should know that for malpractice to occur, six elements should be present.
Which of the following is not an element of malpractice?
*

1/1

a. Duty
b. Modifiability

c. Breach of duty
d. Injury

70. A patient asks the nurse what virtual office visit is. She response correctly by saying:
*

1/1

a. The nurse communicates with a patient through electronic mail.


b. Patients use their smartphones to consult with the health care providers through video.

c. Patients download a software/application in which she will consult an artificial intelligence


equipped with medical knowledge.
d. A hologram of the health care provider is provided so that the patient and nurse can communicate
virtually.
71. The nurse is updating herself in the current trend in health care when she read about
nursing informatics. Which of the following refers to nursing informatics?
*

0/1

a. It is the science of using nursing knowledge to create applications and computer software.
b. The technology in which nurses are informed and educated using computers and software.
c. It is the science of information technology applied to computer systems.

d. Science of using computer information systems in the practice of nursing.


Correct answer
d. Science of using computer information systems in the practice of nursing.

Situation

Nurse Harry is a staff nurse in the otorhinolaryngology ward in Hospital Z. The following questions
apply:

72. Upon assessment of the patient’s hearing acuity, the nurse uses Rinne test. The patient
reports that she hears bone-conducted sound longer in the affected ear than the normal ear.
What can the nurse conclude from the patient’s response?
*

0/1

a. The affected ear has sensorineural hearing loss.

b. The affected ear has conductive hearing loss.


c. The affected ear is recuperating from the damage that occurred.
d. This is a normal finding.
Correct answer
b. The affected ear has conductive hearing loss.

73. A patient was prescribed with hearing aid and has been using it for a week. Which of
the following conditions does the hearing aid problem of whistling noise indicate?
*

0/1

a. Cerumen in ear
b. Dead batteries
c. Loose ear mold
d. Ear infection
Correct answer
c. Loose ear mold

Situation
Patient KZ was admitted with a primary impression of leg fracture and was advised to undergo
surgery. You are the nurse caring for KZ.

74. Which of these would you expect in the affected leg?


*

0/1

a. Lengthening, circumduction

b. Warmth on affected side, bent appearance, swelling


c. Involuntary flexion, dorsiflexion, bleeding
d. Shortening, adduction, external rotation
Correct answer
d. Shortening, adduction, external rotation

75. Buck’s traction was ordered for KZ before she undergoes surgery. This
intervention aims to:
*

0/1

a. Realign the fractured bone


b. Promote good neurovascular status

c. Decrease muscle spasticity


d. Prevent infection from occurring
Correct answer
c. Decrease muscle spasticity

76. How can you maintain counter-traction in a Buck’s traction?


*
0/1

a. Place KZ in supine position


b. The affected leg is flexed every hour.
c. Encourage KZ to maintain High-fowlers position at most times

d. Place weights on the head of the bed with same weight as on the foot of bed
Correct answer
a. Place KZ in supine position

77. Which of these actions is an appropriate intervention for patients on skin traction?
*

1/1

a. Assess for neurovascular status q24 hours.


b. Turn patient q2h to the unaffected side

c. Provide pin site care q 16 hours.


d. Remove the traction weights whenever the patient is uncomfortable.

78. Which is the most important nursing action in caring for KZ with Buck’s traction?
*

0/1

a. Provide water to hydrate the patient adequately.

b. Keep the rope touching the foot of the bed.


c. Assessing the skin under the traction at least every 4 hours
d. Traction can be interrupted if patient will void.
Correct answer
c. Assessing the skin under the traction at least every 4 hours

Situation
A 21-year old male is admitted to the burn unit of X hospital. He sustained burns on the chest,
abdomen, right arm and right leg.
79. The nurse assigned to his care anticipates that the client would be particularly
susceptible to which of the following fluid and electrolyte imbalances during the emergent
phase of burn case.
*

0/1

a. Metabolic acidosis
b. Hypernatremia
c. Hypokalemia

d. Metabolic alkalosis
Correct answer
a. Metabolic acidosis

80. The nurse assesses the client for fluid shifting. During the emergent phase of a burn
injury, shifts occur due to fluid moving from the_______________.
*

0/1

a. Extracellular to intracellular space.


b. Intracellular to extracellular space.

c. Vascular to interstitial space.


d. Interstitial to vascular space
Correct answer
c. Vascular to interstitial space.

81. The nurse understands that the fluid shift results from an increase in
the_____________.:
*

0/1

a. Total volume of intravascular plasma

b. Total volume of circulating whole blood


c. Permeability of capillary walls
d. Permeability of the kidney tubules
Correct answer
c. Permeability of capillary walls

82. The client receives fluid resuscitation therapy. The nurse adjusts the infusion rate by
evaluating the client’s __________:
*

0/1

a. Hourly urine output


b. Daily body weight

c. Hourly urine specific gravity


d. Hourly body temperature
Correct answer
a. Hourly urine output

83. The client receives total parenteral nutrition (TPN). The nurse understands this therapy
will help the client__________.
*

0/1

a. Provide adequate nutrition

b. Ensure adequate caloric and protein intake


c. Correct water and electrolyte imbalances
d. Allow the gastrointestinal tract to rest
Correct answer
b. Ensure adequate caloric and protein intake

Situation
Mr. Arvin is a nurse supervisor of three departments in hospital Baby Shark. She attends an
orientation seminar on hospital records management.

84. Mr. Arvin understands that good client care relies on good record keeping. Which of the
following is NOT a purpose of hospital record keeping?
*

0/1
a. Records provide evidence of a hospital’s accountability.
b. Records are a key source of data for medical research or statistical reports.

c. Records provide data on health information system.


d. Records provide personal information on the physicians and nurses caring for the clients.
Correct answer
d. Records provide personal information on the physicians and nurses caring for the clients.

85. Mr. Arvin is aware that when a client is readmitted to a hospital, the client’s file is
retrieved from the _______________.
*

0/1

a. physician’s file
b. civil service file

c. master patient index file


d. hospital library record file
Correct answer
c. master patient index file

86. Mr. Arvin is aware that when a client is discharged or dies, the following details should
be entered in the client’s record which is the_______________.
*

0/1

a. Final diagnosis
b. Outcomes classification
c. Educational attainment

d. Religion
Correct answer
a. Final diagnosis

87. The following statements are true about patients and hospital records EXCEPT:
*

0/1

a. Confidential records must be protected against loss, damage , unauthorized access, modification
and disclosure
b. Patients have the right to confidential treatment of information they provide to health professional
c. Health records are the property of community where the patient is treated
d. Hospital records maybe released without the patient’s consent when required in investigation for
serious criminal offenses

Correct answer
c. Health records are the property of community where the patient is treated

Situation
Mr. Glenn is a director of the critical care unit of hospital Xavier. She utilizes the nursing process to
communicate care to the client.

88. He is called to the bedside of a client who is scheduled to have laparoscopic


cholecystectomy. The client’s pulse is slightly irregular. Mr. Glenn confers with the primary
nurse regarding the interventions considering the client’s condition, which step of the nursing
process is Mr. Glenn?
*

0/1

a. Implementation
b. Evaluation

c. Planning
d. Assessment
Correct answer
c. Planning

89. Mr. Glenn calls for a conference with the staff members who are attending to the client.
They decide to obtain a 12-lead ECG for a more definitive picture. They conclude that the
client has no serious cardiac or pulmonary problems. Which step of the nursing is in effect
in this situation?
*

0/1
a. Nursing diagnosis

b. Assessment
c. Evaluation
d. Planning
Correct answer
b. Assessment

90. Ms. Glenn consults with the attending physician and the anesthesiologist. She advises
the primary nurse to proceed with the preparations and to remain alert for any adverse
symptoms. Which step of the nursing process is this?
*

1/1

a. Assessment
b. Nursing diagnosis
c. Planning

d. Evaluation

91. Mr. Glenn confers with the client’s primary nurse the following morning. Together they
determine that the client is ready for surgery. This step of the nursing process is:
*

0/1

a. Evaluation
b. Planning
c. Nursing diagnosis
d. Assessment

Correct answer
a. Evaluation

92. Mr. Glenn applies the human relations approach in this situation. He is aware that the
key to productivity is _________________.
*
0/1

a. the degree of independence allowed


b. meeting the objectives of the critical care unit
c. Firm control of the situation

d. the behavior of people under direction


Correct answer
a. the degree of independence allowed

Situation
The nurse in the emergency department admits a 45-year old female for vomiting blood. According to
a family member who accompanied the client, the client had gastric ulcer for a several years. The
nurse assesses that the client is in shock.

93. Which of the following assessment findings indicate hypovolemic shock?


*

0/1

a. Systolic blood pressure is less than 90 mmHg.


b. Pupils are unequally dilated.
c. Respiratory rate is more than 30 breaths per minute.
d. Pulse is less than 60 beats per minute

Correct answer
c. Respiratory rate is more than 30 breaths per minute.

94. In the early stages of shock, the nurse expects the result of arterial blood gas (ABG)
analysis to indicate which of the following conditions ____________:
*

0/1

a. Respiratory alkalosis
b. Respiratory acidosis
c. Metabolic alkalosis

d. Metabolic acidosis
Correct answer
a. Respiratory alkalosis

95. The physician orders intravenous infusion of packed red blood cells and normal saline
solutions. The nurse assesses the client for which of the following _____________:
*

0/1

a. Hypovolemia
b. Anaphylactic reaction
c. Altered level of consciousness

d. Pain
Correct answer
b. Anaphylactic reaction

96. The nurse understands that the best indication that fluid replacement for the client in
hypovolemic shock is adequate is when the ___________:
*

0/1

a. Systolic blood pressure is above 110 mmHg.


b. Diastolic blood pressure is above 90 mmHg.

c. Urine output of 20- 30 mL/Hour.


d. Urine output is greater than 30 mL/Hour.
Correct answer
d. Urine output is greater than 30 mL/Hour.

97. The physician schedules the client for surgery within six hours. The nurse minimizes the
anxiety of the client by answering the client’s questions regarding the surgery in calm
manner, keeps the client warm, advises the client to be on bed rest and dims the lights in the
room. The reason for these interventions is to ________:
*

0/1

a. Increase comfort of the client and her family.


b. Minimize oxygen consumption.
c. Prevent infection.
d. Stabilize fluid and electrolyte balance.
Correct answer
a. Increase comfort of the client and her family.

Situation
You are a nurse working in a medical ward in Cornelius Medical Center. The following questions
apply:

98. A patient who suffered from multiple trauma in the street was seen by the nurse. She
responds initially by:
*

1/1

a. Positioning properly.
b. Opening the airway.

c. Providing oxygen.
d. Calling emergency medical services.

99. Diagnostic cardiac catheterization was performed to a client to determine presence of


coronary artery disease (CAD). The following are the correct nursing responsibility post-
cardiac catheterization except?
*

0/1

a. Observe the catheter access site for bleeding or hematoma formation.


b. Bed rest must be maintained for 2-6 hours after procedure.

c. Instruct to perform exercise as allowed by physician.


d. Monitor for signs of hypertension and tachycardia.
Correct answer
d. Monitor for signs of hypertension and tachycardia.
100. The patient was admitted to the trauma ward and is being assessed by the nurse. Which
of the following parameters must the nurse observe when using Modified Champion Trauma
Scoring?
i. Glasgow Coma Scale
ii. Respiratory rate
iii. Systolic blood pressure
iv. Diastolic blood pressure
*

0/1

a. i, ii, iv

b. i, iii, iv
c. i, ii, iii
d. ii, iii, iv
Correct answer
c. i, ii, iii

RECALLS 3 - NP4
Total points74/100

1. Which of the following is the best indirect method of measuring blood pressure?
*

1/1

a. Palpatory blood pressure


b. Aneroid sphygmomanometer

c. Pulse oximeter
d. Intra-arterial catheter

2. According to the standards of JNC 8, hypertension stage 1 is described as:


*

1/1

a. Systolic BP of 120-139mmHg, diastolic BP of 80-89mmHg


b. Systolic BP of 120-149mmHg, diastolic BP of 90-99mmHg
c. Systolic BP of 140-159mmHg, diastolic BP of 90-99mmHg

d. Systolic Bp of 140-149, diastolic BP of 100-110mmHg

3. A patient was ordered to have his EKG reading done. As the nurse, you know that the
electrocardiogram (EKG) is performed to assess:
*

0/1

a. Heart chambers and heart valves


b. Hypertrophy, infarction, axis deviation
c. Dysrhythmias, mitral stenosis, electrical conduction

d. Coronary vessels and artery disease


Correct answer
b. Hypertrophy, infarction, axis deviation

4. During a disaster, which principle is integral in decision-making and providing health care
to those who are victims?
*

1/1

a. Beneficence
b. Non-maleficence
c. Utilitarianism

d. Paternalism

5. A patient admitted to the hospital with myocardial infarction develops severe pulmonary
edema. Which of the following symptoms should the nurse expect the patient to exhibit?
*

1/1

a. Slow, deep respirations


b. Stridor
c. Bradycardia
d. Air hunger

6. A child is seen in the emergency department for scarlet fever. Which of the following
descriptions of scarlet fever is NOT correct?
*

1/1

a. Scarlet fever is caused by infection with group A Streptococcus bacteria


b. “Strawberry tongue” is a characteristic sign.
c. Petechiae occur on the soft palate.

d. The pharynx is red and swollen.

Situation
You are a staff nurse in a psychiatric unit. You are taking care of Mr. K. who is suffering from heroin-
addiction with suspected complications of HIV.

7. Loh asked you what HIV seropositivity means. Your answer will be:
*

1/1

a. An infected person with HIV is capable of transmitting the virus to sexual partner.

b. An infected person can donate blood after five years.


c. The person tested is not infectious.
d. With appropriate medication, the infected person will be no longer infectious after two months.

8. In your morning rounds, you noticed in Mr. K. the presence of cough, shortness of breath
and tachypnea. Which of the opportunistic infection is probably causing these
manifestations?
*

1/1

a. Toxoplasmgondii
b. Cytomegalo virus
c. Cryptococcus neoformans
d. Pneumocystis jirovecii
9. Which of the following are the most possible cause of HIV in Mr. K’s case?
*

0/1

a. Unprotected sex with his fiancée


b. Exposure to infected body fluid.
c. Unprotected anal intercourse.

d. His heroin addiction


Correct answer
d. His heroin addiction

10. Another day, Mr. K. was visited by her girlfriend Loh, who admitted she had been having
sex with him. In describing risky sexual practices, which of the following you will tell Loh is
not a risk factor?
*

0/1

a. Mucosal exposure such as splashing the eyes or mouth

b. Anal intercourse is the primary way in spreading HIV


c. HIV can be transmitted during sexual intercourse from an infected partner.
d. Oral sex is considered risky.
Correct answer
b. Anal intercourse is the primary way in spreading HIV

11. Mr. K tested positive for HIV. Which of the following explanation can you give him?
*

0/1

a. “Antibodies to the AIDS virus are present in the blood.”


b. “This means that you will not develop AIDS in the future.”
c. “You have been diagnosed with AIDS.”
d. “At this point, AIDS virus is not active in your blood.”
Correct answer
a. “Antibodies to the AIDS virus are present in the blood.”

Situation
A 60-year old male is admitted to the oncology unit. According to the client, he felt a growth during a
routine digital prostate examination. He complains of pain on urination and frequent urination.

12. The nurse understands that the function of the prostate gland is primarily to ______;
*

1/1

a. Regulate the acidity and alkalinity environment for proper sperm development.
b. Produce a secretion that aids the nourishment and passage of sperm.

c. Secrete a hormone that stimulates the production and maturation of sperm.


d. Store undeveloped sperm before ejaculation.

13. The nurse analyzes the laboratory values and notes that the serum phosphate level is
elevated. This finding indicates which of the following:
*

1/1

a. It confirms the diagnosis of prostate cancer.


b. The progression or regression of prostate cancer.
c. The likelihood of metastasis to the bones.

d. There are complications associated with cancer.

14. The nurse knows that hormone therapy is the mode of treatment for a client with prostate
cancer. The goal of this form of treatment is to ______:
*

1/1

a. Limit the amount of circulating androgens.

b. Increase prostaglandin level.


c. Increase the amount of circulating androgens.
d. Increase testosterone level.

Situation
The nurse cares for a female client who is terminally ill and is experiencing pain.

15. The nurse prepares a care plan for the client. The overall goal for the client is ________.
The client will:
*

1/1

a. Achieve control of pain and discomfort.

b. Receive adequate cerebral oxygenation and perfusion.


c. Be free from infection.
d. Receive life sustaining food and liquids.

16. The nurse is aware of the document that expresses a client’s wish for life sustaining
treatment in the event of terminal illness or permanent unconsciousness. This document is the
______;
*

1/1

a. No-code order
b. Durable power of attorney
c. Living will

d. Last will and testament

17. The client nears death and requests that no medication be given that would cause a loss of
consciousness, including pain medication. The nurse would promote the best end-of-life care
for the client by which of the following?
*

1/1

a. Discuss the request of the dying client with family members and respect their wishes.
b. Comfort is the highest priority in this situation so give medications as ordered.
c. Respect the client’s wishes and withhold pain medications and other medications ordered.

d. Be compassionate and give half of dose of the medication ordered.

18. Which of the following statement is TRUE about terminally ill clients?
*

1/1

a. Terminally ill clients require minimum physical care.


b. Health care personnel do not understand their own feelings about death and dying therefore they
avoid caring for terminally clients.
c. Terminally ill clients have the right to die with dignity.

d. Terminally ill client’s experiences pain most of the time.

19. The dying clients wishes to donate her eyes after she dies. Which of the following
statements is NOT TRUE about organ donation?
*

0/1

a. Any individual, at least 15 years old of age and of a sound mind may donate a part of his body to
take the effect after transplantation needed by the recipient.
b. Sharing of human organs or tissues shall be made only through exchange programs duly approved
by the Department of Health.
c. The choice to donate an organ must be a written document.
d. Laws do not require the consent of a family members to retrieve organs if the donor has expressed
his last wish to donate.

Correct answer
a. Any individual, at least 15 years old of age and of a sound mind may donate a part of his body to
take the effect after transplantation needed by the recipient.

Situation
The charge nurse in the medical unit updates her knowledge on nursing diagnosis. She reviews the
terms used to describe clinical adjustment.

20. A nursing diagnosis formulated when there is insufficient evidence to support the
presence of the problem but the nurse believes the problem is highly probable and wants to
collect more data is a/an _________:
*

1/1

a. Risk nursing diagnosis


b. Possible nursing diagnosis

c. Actual nursing diagnosis


d. Wellness nursing diagnosis

21. Which of the following statements is a POSSIBLE nursing diagnosis?


*

1/1

a. Constipation related to decreased activity and fluid intake


b. Potential for Enhanced Spiritual Well Being
c. Possible Self-Care Deficit: grooming related to fatigue and muscular weakness

d. Risk for Activity Intolerance related to prolonged bed rest

22. Which of the following is a RISK nursing diagnosis?


*

1/1

a. Potential for Enhanced Spiritual Well Being


b. Possible Self-Care Deficit; grooming related to fatigue and muscular weakness
c. Risk for Activity Intolerance related to prolonged bed rest

d. Constipation related to decreased activity and fluid intake

Situation
The nurse provides health education to a group of adolescents about pelvic inflammatory disease
(PID).

23. The nurse explains that prevention of PID in adolescents is important due to which of the
following reasons. PID ________:
*
1/1

a. can have devastating effects on the reproductive tract of affected adolescents.

b. is easily prevented by compliance to any form of contraception


c. can cause life-threating defects in infants born to affected adolescents.
d. Is easily prevented by proper personal hygiene

24. The nurse explains to the group of adolescents that the most common cause of PID is
_______:
*

1/1

a. Tuberculosis bacilli
b. Gonorrhea

c. Staphylococcus
d. Streptococcus

25. The nurse further explains that PID presents the following signs and symptoms, which
the adolescent should be aware of:
*

1/1

a. A hard painless, red and defined lesion on the genital area.


b. Small vesicles on the genital area with itching.
c. Lower abdominal pain and urinary tract infection.

d. Cervical discharge with redness and edema.

26. Which of the following statements is true when teaching adolescents about gonorrhea?
*

1/1

a. Gonorrhea may be contracted through contact with a contaminated toilet seat.


b. The infectious agent for gonorrhea is Neisseria gonorrheae.
c. Gonorrhea is most often treated by multidose of administration of penicillin.
d. Treatment of sexual partners is the priority of treatment.

27. The nurse further explains to the adolescents that gonorrhea is highly infectious and it
____:
*

1/1

a. Is limited to the external genitalia.


b. Can lead to sterility.

c. Is easily treated.
d. Occurs rarely among adolescents.

28. The following are some of the steps to using a male condom. Which comes first?
*

1/1

a. Pinch the top of the condom to remove any air.

b. Dispose of condom safely and hygienically.


c. Roll the condom down over the penis.
d. Apply a water-based lubricant over the penis.

Situation
A 45-year old female, married with two children, is admitted in the medical department with a
diagnosis of osteoarthritis. The nurse assists in the care of this client.

29. Which of the following signs and symptoms should the nurse correlate with a diagnosis of
osteoarthritis?
*

1/1

a. Erythema and edema over the affected joints


b. Joints stiffness that decrease with activity
c. Anorexia and weight loss
d. Fever and malaise

30. Which of the following factors would MOST likely increase the joints symptoms of
osteoarthritis?
*

1/1

a. Emotional stress
b. Obesity

c. History of smoking
d. Alcohol abuse

Situation
Venice, is an autistic child who loves to bang her head. One day, while head banging, she
unintentionally hit the wall and briefly loses consciousness.

31. Venice is brought into the emergency department of Malan-dee Medical Center after
suffering a head injury. The first action by the nurse is to determine the Venice’s:
*

1/1

a. Level of consciousness
b. Pulse and blood pressure
c. Respiratory rate and depth

d. Ability to move extremities

32. The nurse anticipates detecting the occurrence of what common complication of head
injury?
*

0/1

a. Intracranial haemorrhage
b. Diabetes insipidus
c. Diabetes mellitus
d. Basilar skull fracture
Correct answer
b. Diabetes insipidus

33. The nurse is aware that she should monitor Venice’s?


*

0/1

a. Pulse rate

b. Temperature
c. Urine output
d. Oxygen saturation
Correct answer
c. Urine output

34. Venice is ordered to receive desmopressin (DDAVP) for management her condition. The
nurse should check which of the following measurements to determine the effectiveness of
this medication?
*

1/1

a. Daily weight

b. Temperature
c. Apical heart rate
d. Pupillary response

35. The nurse knows that which of the following conditions may alter the effectiveness of
DDAVP (Noctiva)?
*

0/1
a. Increased oral secretions
b. Arterial obstruction
c. Nasal congestion
d. Obesity

Correct answer
c. Nasal congestion

SITUATION
A 30-year old client named Gardo Versosa is admitted to Philippine Lung Center due to sudden onset
of chest pain and dyspnea. He has no history of respiratory disease but had complete femur fracture 3
days ago. The following questions apply.

36. He is diagnosed with pulmonary embolism. The nurse immediately implements which
expected prescription for this client?
*

1/1

a. Semi-Fowler’s position, oxygen, and morphine sulfate (IV)

b. Supine position, oxygen, and meperidine hydrochloride (Demerol) [IM]


c. High Fowler’s position, oxygen, and meperidine hydrochloride (Demerol) [IV]
d. High Fowler’s position, oxygen, and two tablets of acetaminophen with codeine (Tylenol #3)

37. The doctor ordered ABG analysis for Gardo Versosa. The nurse is now sending the ABG
specimen to the laboratory for analysis. Which of the following pieces of information should
the nurse write on the laboratory requisition?

1 Ventilator settings
2 A list of client allergies
3 The client’s temperature
4 The date and time the specimen was drawn
5 Any supplemental oxygen the client is receiving
6 Extremity from which the specimen was obtained
*

0/1

a. 1, 3, 5, 6
b. 1, 2, 4, 5
c. 1, 2, 3, 4

d. 1, 3, 4, 5
Correct answer
d. 1, 3, 4, 5

38. The nurse is now inserting an oropharyngeal airway to Gardo Versosa. The nurse plans to
use which correct insertion procedure?
*

0/1

a. Flex the client’s neck

b. Leave any dentures in place


c. Suction the client’s mouth once per shift
d. Insert the airway with the tip pointed upward
Correct answer
d. Insert the airway with the tip pointed upward

39. Gardo Versosa is now intubated and receiving mechanical ventilation. The physician has
added 7 cm of positive end expiratory pressure (PEEP) to the ventilator settings of the client.
The nurse assesses for which of the following expected but adverse effects of PEEP?
*

0/1

a. Decreased peak pressure on the ventilator


b. Increased temperature from 98OF to 100OF rectally

c. Decreased heart rate from 78 to 64 beats per minute


d. Systolic blood pressure decrease from 122 to 98 mm Hg
Correct answer
d. Systolic blood pressure decrease from 122 to 98 mm Hg

SITUATION
Dina is a 50-year old obese patient. She admits that her self-esteem has been progressively becoming
low as her body size increases. She also claimed that her performance in her work has already been
impaired. She electively subjects herself to a bariatric surgery.
40. Rapid emptying of gastric contents into the small intestine may occur postoperatively
due to gastric resection. Dina is at risk for developing dumping syndrome. The nurse
monitors her for:
*

0/1

a. Dizziness
b. Bradycardia
c. Constipation
d. Extreme thirst

Correct answer
a. Dizziness

41. The nurse places Dina in which best position to prevent occurrence of dumping
syndrome?
*

0/1

a. Supine
b. Trendelenburg
c. Fowler’s

d. Prone
Correct answer
a. Supine

42. Dina now ready to resume diet since she now has normal bowel sounds. To minimize
complications from eating, the nurse teaches her to do which of the following?

1. Lying down after eating


2. Eating a diet high in protein
3. Eating a diet low in protein
4. Eating six small meals per day
5. Eating concentrated sweets between meals
*

1/1

a. 1, 3, 5
b. 1, 2, 5
c. 1, 2, 4

d. 1, 3, 4

43. In preventing dumping syndrome, which additional instruction should be provided to


Dyosa Lande?
*

0/1

a. Ambulate following a meal


b. Eat high carbohydrate foods
c. Limit the fluids taken with meals
d. Sit in a high Fowler’s positions during meals

Correct answer
c. Limit the fluids taken with meals

SITUATION
Patients with varying diseases always receive medications for them to recuperate. As part of the
dependent functions of a nurse, knowledge in pharmacology is one of the most essential competencies
that she should possess, to be aware when to verify a doctor’s medication order, and to know the side
effects, adverse effects, therapeutic level and nursing consideration for each drug.

44. A nurse in the neuro ward is reviewing the results of a client Rey’s phenytoin (Dilantin)
level that was drawn that morning. The nurse determines he had a therapeutic drug level if
the result was:
*

0/1

a. 3 mcg/mL
b. 8 mcg/mL

c. 15 mcg/mL
d. 24 mcg/mL
Correct answer
c. 15 mcg/mL
45. Client Cyrus Mc Leo has begun medication therapy with Betaxolol (Kerlone). The nurse
determines he is experiencing the intended effect of therapy if which of the following is
noted?
*

1/1

a. Edema present at 3+
b. Weight loss of 5 pounds
c. Pulse rate increased from 58 to 74 beats/ min
d. Blood pressure decreased from 142/94 mm Hg to 128/82 mm Hg

46.The nurse has taught another client named Fedilyn with asthma, who is taking a xanthine
bronchodilator about beverages to avoid. The nurse determines that the client understands the
information if the client chooses which of the following beverages from the dietary menu?
*

1/1

a. Cola
b. Coffee
c. Chocolate milk
d. Cranberry juice

47. Client Roma is ordered to start on Glipizide once daily. The nurse observes for which of
the following intended effect of this medication?
*

1/1

a. Weight loss
b. Resolution of infection
c. Decreased blood glucose

d. Decreased blood pressure


48. Client Jahziel, a toddler is hospitalized for acetaminophen (Tylenol) overdose. The
nurse prepares to administer which specific antidote for this medication overdose?
*

1/1

a. Vitamin K
b. Protamine sulphate
c. Acetylcysteine (Mucomyst)

d. Naloxone hydrochloride (Narcan)

SITUATION
A nurse working in the hema ward of Bloody hospital for 3 years is assigned to care for pediatric
patients with hereditary condition. One of her patients is named Newden, diagnosed with B-
Thalassemia.

49. The nurse is presenting a clinical conference and discusses the cause of B-Thalassemia.
The nurse informs her audience that the child at greatest risk of developing this disorder is:
*

1/1

a. A child of Mexican descent


b. A child of Mediterranean descent

c. A child of Asian descent


d. A child of American descent

50. he nurse added that in B-Thalassemia, which family history may be present in the
development of the condition?
*

1/1

a. Autosomal dominant disorder


b. Autosomal recessive disorder

c. Y-linked genetic disorder


d. X-linked genetic disorder
51. The nurse presented assessment data she gathered from her patient. She emphasized that
her patient has greenish-yellow skin tone and severe anemia requiring transfusion support to
sustain life. She identifies this type of B-Thalassemia as:
*

1/1

a. Thalassemia major

b. Thalassemia intermedia
c. Thalassemia trait
d. Thalassemia minor

52. Based from the patient’s manifestations, severe anemia in B-Thalassemia is also known
as:
*

1/1

a. Kleihauer-Betke’s anemia
b. Charcot’s anemia
c. Coumb’s anemia
d. Cooley’s anemia

53. The patient is receiving long-term blood transfusion therapy for the treatment of his
disorder. Chelation therapy is prescribed to prevent organ damage from the presence of too
much iron in the body as a result of the transfusions. The nurse correctly anticipates the
medication used for chelation therapy for the patient which is:
*

1/1

a. Naloxone
b. Calcium Disodium Edetate
c. Deferoxamine

d. Protamine sulfate
54. The nurse is caring for another patient with a hereditary bleeding disorder. The nurse
noted that the patient has increased tendency to bleed from mucous membranes. Most
probably, the physician’s medical diagnosis for this patient would be:
*

1/1

a. Christmas disease
b. Classic hemophilia
c. Von Willebrand disease

d. B-Thalassemia

Situation
Mrs. Lima, a 47-year old married woman with four children, went to the hospital because of joint
pain. Upon reviewing her medical history, the nurse discovers that she was diagnosed with
osteoarthritis.

55. Nurse Kathryn was assigned to care for this patient. She is aware that osteoarthritis is
not associated with the following signs and symptoms:
*

0/1

a. Edema over the affected joints


b. Stiffness is decreased with movement.
c. Pain
d. Limitations in range-of-motion

Correct answer
a. Edema over the affected joints

56. A comprehensive physical assessment and health history was taken by Nurse Kathryn.
She was able to take note of various risk factors present from the patient's lifestyle. Nurse
Kathryn knows that among the following, the factor that most likely aggravates Mrs. Lima's
symptoms is:
*

1/1

a. Recent leg fracture


b. Working as a corporate president for 10 years
c. Weight of 75kg and height of 165cm

d. Dehydration

57. The nurse encourages the patient to have her target weight of:
*

1/1

a. 49.8 kg
b. 61.3 kg
c. 58.5 kg

d. 52 kg

58. Nurse Kathryn is aware that osteoarthritis is a "wear-and-tear" disease. She expects that
the joints most likely affected in this condition are the:
*

1/1

a. Hips and knees

b. Tibia and fibula


c. Humerus and radius
d. Thoracic spine

59. As part of health teaching, Nurse Kathryn instructs Mrs. Lima that to effectively decrease
joint pain and stiffness before starting her daily activities, she should not do the following
excluding:
*

0/1

a. Decrease carbohydrates and protein, and increase more fat in diet.

b. Perform range-of-motion exercises and apply liniment to affected joints


c. Administer codeine when pain is exceedingly high.
d. Apply cold compress to affected joints.
Correct answer
b. Perform range-of-motion exercises and apply liniment to affected joints

60. Being the patient's primary nurse, Nurse Kathryn collaborates with the healthcare team,
especially with the physical therapist. The physical therapist recommended that Mrs. Lima
undergo a regimen of rest, exercise and physical therapy. Nurse Kathryn explains to the
patient that this regimen will:
*

1/1

a. Help patient cure the disease


b. To reduce the inflammation due to the disease process.
c. To restore her abilities she had when she was younger
d. Prevent the crippling effect of osteoarthritis

Situation
: Nurse Rey receives a call informing the ER department of Hospital X that a bus fell off a cliff.
Several passengers were severely injured and will be brought to the hospital. Nurse Rey informs her
co-staff and starts preparing to cater the injured passengers.

61. Being an ER nurse, Rey knows that triaging is a skill that she must master. She is aware
that triaging is performed in order to:
*

1/1

a. Prioritize those who are capable of paying


b. Give way to those who can be treated easier, followed by those critically-ill
c. Sort patients who will likely survive or not.
d. Direct all available resources to the most critically ill patients

62. ER nurses in Hospital X uses the 5-level of emergency severity index (ESI). Nurse Rey
received a patient who is conscious but disoriented and complains of pain on the occipital
part of his head. Nurse Rey knows that this patient is classified under:
*

0/1

a. ES 1
b. ES 2
c. ES 3

d. ES 4
Correct answer
b. ES 2

63. Nurses are expected to be competent in prioritizing. In performing an emergency


assessment for the injured passengers, Nurse Rey knows that she should prioritize:
*

1/1

a. Breathing assistance
b. Stable hemodynamics
c. Airway clearance

d. Infection control

64. Nurse Rey receives and assesses an injured patient. He referred to the resident-on-duty
that the patient is possibly experiencing hypovolemic shock. Which of the findings best
supports this?
*

1/1

a. PR-112bpm, RR-27cpm, BP-80/60mmHg

b. PR-55bpm, RR-30cpm, BP-130/80mmHg


c. PR-53bpm, RR-11cpm, BP-140/90mmHg
d. PR-98bpm, RR-18cpm, BP-100/60mmHg

65. Another patient rushed into the hospital sustained a concussion during the accident. Nurse
Diosrey assesses that he opens his eyes spontaneously and obeys commands. After
assessment, she gives him a GCS score of 14. Given the circumstances, which among the
following is the priority nursing diagnosis?
*

0/1
a. Acute pain

b. Risk for infection


c. Impaired physical mobility
d. Risk for injury
Correct answer
d. Risk for injury

Situation
A 46-year-old male patient is brought to the emergency department with chief complaint of cervical
lymphadenopathy which started a week prior to consult. The primary impression is Hodgkin’s
Lymphoma.

66. In the later course of the condition, which of these are expected manifestations?
*

1/1

a. Fever, night sweats, weight loss

b. Diarrhea, abdominal pain, hematochezia


c. Bleeding tendencies, petechiae, melena
d. Ascites, jaundice, anorexia

67. The nurse assigned to the patient notes knowledge deficit on self-care and risk prevention.
What should the nurse do to challenge this noted nursing diagnosis?
*

1/1

a. Document the nursing diagnosis with the observed cues.


b. Plan a health teaching

c. Start a meeting with patient’s SOs


d. Refer to the physician.
68. The patient asks the nurse, “Why am I given four chemotherapeutic drugs all at once over
a prolonged duration?” The nurse responds with the correct knowledge that:
*

1/1

a. You may not respond to other drugs. Four drugs ensures that the cancer cells are killed.
b. If you have resistance to one drug, the other drugs will do the chemotherapeutic action.
c. Development to Stages III and IV is prevented.
d. The combination drugs have different and synergistic actions.

69. Which laboratory finding of a patient receiving chemotherapeutic drug will alert the nurse
to contact the physician?
*

1/1

a. RBC count of 5 million/uL


b. WBC count of 1700/uL

c. Hemoglobin of 14 mg/dL
d. Serum potassium of 5 mE

70. What will be the most appropriate nursing diagnosis for patients undergoing
chemotherapy?
*

1/1

a. Risk for injury r/t radioactive damage to epidermal and dermal layers
b. Fatigue r/t decreased oxygen-carrying capacity
c. Risk for infection r/t immunosuppressive effects of chemotherapy

d. Impaired tissue integrity r/t decreased tissue perfusion

Situation
Archie has been wearing eyeglasses since he was 5 years old. When he turned 18, he wanted to
dispose of the eyeglasses and started wearing long-wearing contact lenses. Before his 24th birthday,
he was rushed into the emergency department because of severe eye pain. After assessment, he was
diagnosed to have corneal ulcer.
71. Which of the following affects the ability of the eye to clearly focus? A change in
the_______.
*

1/1

a. Stroma
b. Sensory cells of the retina
c. curvature of the cornea

d. epithelium

72. Which of the following is the predisposing factor for Marjorie condition?
*

1/1

a. Bacterial infection
b. Prolonged misuse of contact lenses

c. Malnutrition
d. Viral infection

73. Corneal ulcers are considered medical emergencies. Which of the following nursing
actions will be your PRIORITY?
*

0/1

a. Remove the contact lens


b. Prompt referral to the ophthalmologist for treatment

c. Administer eyedrops
d. Pressure dressing applied to both eyes for comfort
Correct answer
a. Remove the contact lens

Situation
Nurse Alfie works in the oncology unit. She takes care of cancer patients in pain. She is aware that
cancer pain management is one of her responsibility.
74. Nurse Alfie collaborates with the physician in the development of a drug regimen for the
clients. Which of the following medications is least preferred in the treatment of cancer pain?
*

0/1

a. Morphine
b. Acetaminophen (Tylenol)
c. Meperidine (Demerol)

d. Hydrocodone
Correct answer
b. Acetaminophen (Tylenol)

75. When titrating a drug for the client in pain, which of the following action is MOST
appropriate?
*

1/1

a. Ask the physician to include a medication order for breakthrough pain.


b. Follow the physician’s order for the first 24 hours.
c. Reassess the client every 8 hours for drug effectiveness.
d. Seek a new order after 2 doses that do not achieve a tolerable level of pain relief.

76. One of the clients experience severe, intractable pain and complains that the pain
medication is not working for him. Which of the following actions is MOST appropriate for
Nurse Alfie?
*

1/1

a. Suggest to the client to try deep breathing to cope with the pain.
b. Explore the nature of the pain and encourage the client to perceive it in a different way.
c. Support the client emotionally and tell him he will receive the next dose of medication as soon as
possible.
d. Refer the client to the attending physician immediately and report that the pain medication is not
providing adequate pain relief.
77. Nurse Alfie assesses a client complaining of acute pain. The MOST appropriate nursing
assessment would include which of the following?
*

1/1

a. The nurses’ impression of clients’ pain.


b. The clients’ pain rating.

c. Nonverbal cues from the client.


d. Pain relief after appropriate nursing interventions.

Situation
Nurse Julia is working as a staff nurse in the neurosciences ward. The following questions apply.

78. A patient who suffered from vehicular accident was intubated due to respiratory arrest. At
which level/s of the spinal cord does the nurse suspect to be affected?
*

0/1

a. C1-C2
b. C3-C4

c. C5
d. C6
Correct answer
a. C1-C2

79. A paraplegic client is for discharge to home. The nurse correctly includes which
instruction for patient education?
*

1/1

a. Follow-up to this clinic if he already feels fine.


b. Maintain on bed rest for first 48 hours for further recovery.
c. Instruct to borrow crutches from a friend so he does not need to buy.
d. Encourage home modification based on patient needs.

80. The nurse who care for a patient with SCI found out that he also has renal calculi. The
nurse knows that patients with SCI are predisposed to developing kidney stones because:
*

0/1

a. Decreased ability to drink water may have led to dehydration, thus concentrating urine particles.

b. Spinal shock from SCI increased serum uric acid levels, causing uric acid stones.
c. SCI and kidney stones have different and separate etiologies.
d. Immobility due to SCI increases calcium resorption.
Correct answer
d. Immobility due to SCI increases calcium resorption.

81. A 5-year old pediatric client was admitted for tuberculous meningitis, and
ventriculoperitoneal shunt (VPS) was surgically applied. The mother asks the nurse why it
was placed. Which of the following is a correct statement by the nurse?
*

1/1

a. The shunt is applied to drain the bacteria from your child’s ventricles.
b. It prevents further crossing of bacteria through the blood brain barrier (BBB).
c. This shunt will provide IV access for the anticipated medications to be given to the child.
d. It gives passageway for the overflow CSF to exit the ventricles.

82. The patient with spinal cord injury and paraplegic asks the primary reason why has to be
turned every 2 hours. The nurse’s best response is:
*

1/1

a. Patient’s back is encouraged to be exposed to air so perspiration is minimized.


b. Turning is a form of exercise for a patient with paralysis from waist down.
c. It prevents erosion of bones due to consequent immobility.
d. Turning prevents prolonged reduced blood flow to skin.

83. Which of the following is not an expected manifestation from a patient with anterior cord
syndrome?
*

1/1

a. Loss of pain sensation


b. Loss of temperature function
c. Loss of touch sense

d. Loss of motor function

84. An infant is brought to the clinic by his mother, who has noticed that he holds his head in
an unusual position and always faces to one side. Which of the following is the most likely
explanation?
*

1/1

a. Torticollis

b. Craniosynostosis
c. Plagiocephaly
d. Hydrocephalus

Situation

Avian flu had an epidemic in 2017 in different parts of Luzon. Nurse Bonnie is instructing
communities about this disease.

85. Bird’s flu virus (H5N6) had an outbreak in the provinces of Nueva Ecija and Pampanga
this 2017. Which of the following is true of the disease?
*

1/1

a. Humans can contract it from infected pigs.


b. Infected birds shed the virus in their saliva, mucous and feces.
c. Cats and dogs can be carriers of the virus.
d. H5N6 least commonly infects aquatic birds.

86. The nurse instructs the communities in Nueva Ecija about prevention and treatment of
bird’s flu. Which of the following is not part of her teaching?
*

1/1

a. A separate isolation facility is provided for those who will contract the disease.

b. The chickens within a specified radius will be culled.


c. Do not transport chickens outside the area of outbreak.
d. Wear appropriate personal protective equipment when working with poultry.

Situation
You are a nurse giving care to patients in the Emergency Department in Hospital Borobudur.

87. During a disaster, which principle is integral in decision-making and providing health
care to those who are victims?
*

1/1

a. Beneficence
b. Non-maleficence
c. Utilitarianism

d. Paternalism

88. A self-employed auto mechanic is diagnosed with carbon monoxide poisoning.


Admission vital signs are blood pressure 90/42 mm Hg, pulse 84/min, respirations 24/min,
and oxygen saturation 94% on room air. What is the nurse's priority action?
*

1/1

a. Administer 5 mg inhaled albuterol nebulizer treatment to decrease inflammatory


bronchoconstriction
b. Administer 100% oxygen using a nonrebreather mask with flow rate of 15 L/min

c. Administer methylprednisolone to decrease lung inflammation from toxic inhalant


d. Titrate oxygen to maintain pulse oximeter saturation of >95%

89. A nurse is assisting in the care of a client who is to be cardioverted. The nurse plans to set
the defibrillator to which to which of the following starting energy range levels, depending on
the specific physician order?
*

1/1

a. 50 to 100 joules
b. 150 to 200 joules

c. 250 to 300 joules


d. 350 to 400 joules

90. A nurse has applied the patch electrodes of an automatic external defibrillator (AED) to
the chest of a client who is pulseless. The defibrillator has interpreted the rhythm to be
ventricular fibrillation. The nurse then:
*

1/1

a. Orders any personnel away from the client, charges the machine, and defibrillates through the
console

b. Performs cardiopulmonary resuscitation (CPR) for 1 minute before defibrillating


c. Charges the machine and immediately pushes the "discharge" buttons on the console
d. Administers rescue breathing during the defibrillation

91. A woman with hypothyroidism asks the nurse why the doctor told her she cannot have a
sedative. The nurse’s response is based on which of the following facts?
*

0/1

a. Sedatives potentiate thyroid replacement medication.


b. Clients with hypothyroidism have increased susceptibility to all sedative drugs.
c. Sedatives will have a paradoxical effect on clients with hypothyroidism.

d. Sedatives would cause fluid retention and hypernatremia.


Correct answer
b. Clients with hypothyroidism have increased susceptibility to all sedative drugs.

92. Nausea and vomiting in which client is of greatest concern to the nurse?
*

0/1

a. Client postoperative ophthalmic surgery


b. Client receiving chemotherapy

c. Client with Ménière disease


d. Client with severe gastroenteritis
Correct answer
a. Client postoperative ophthalmic surgery

Situation
Gerald, a 26-year old construction worker, arrives in the Ear Clinic riding a Honda Motorcycle. He is
wearing a headset with Mp3 player hooked to his belt. Nelson is for hearing assessment.

93. Pure Tone Audiometry is ordered for Gerald. Which of the following does this procedure
measure?
*

1/1

a. Vestibular portion of the auditory nerve


b. Ear canal volume
c. Structure of the cars
d. Hearing acuity

94. Gerald was found to have Mastoiditis. Which of the following ear structure is affected?
*
1/1

a. Tympanic membrane
b. Pinna
c. Eustachian tube
d. Mastoid air cells

95. Which of the following is the most common cause of Mastoiditis?


*

1/1

a. Bone tumor
b. Untreated Otitis Media

c. Meningitis
d. Mastoid diseases

96. Antibiotics have limited use in the actual treatment of Mastoiditis because________.
*

1/1

a. Tissue destruction is extensive


b. It is a long-term treatment
c. Antibiotics do not easily penetrate the infected bony structure of the mastoid

d. Culture has to be done to identify which antibiotic is most effective for the treatment of Mastoiditis

97. Which of the following is the most common treatment for Mastoiditis?
*

1/1

a. Mastoidectomy only
b. Mastoidectomy with tympanoplasty

c. Antibiotics with tympanoplasty


d. Antibiotics

Situation
You are a staff nurse at the Neurology unit of the Pediatric Ward.

98. You have a patient for admission who has been diagnosed with Bacterial Meningitis.
Which of the following type of isolation should you implement?
*

1/1

a. Contact precaution
b. Universal precaution
c. Airborne precaution
d. Droplet precaution

99. The attending physician of Mr M who has Cerebral Palsy and a seizure disorder
prescribed Tegretol-XR for him. Master M has a gastrostomy feeding tube. The medication
prescribed is on the hospital’s “No crush list”. Which of the following you should do in order
to administer the medication?
*

1/1

a. Ask the pharmacist for oral suspension

b. Contact the attending physician to change the order


c. Dissolve the medication in 30 ml. of orange juice
d. Cut the medication into small pieces to be places to be placed in the feeding tube

100. You have been observed by your nurse manager that when you gave the I.V. medication,
you disconnected the flush syringe first and then clamped the intermittent infusion device.
Which of the following would be the most effective way to improve this nursing practice?
*

0/1

a. Post an evidenced-based article on administration of I.V. medication in the neurological unit


b. Send a group e-mail discussing the importance of clamping the device first
c. Create a poster presentation on administration of I.V.
d. Ask each nurse if they are aware that their practice is obsolete
Correct answer
a. Post an evidenced-based article on administration of I.V. medication in the neurological unit

RECALLS 3 - NP5
Total points66/100

1. You have a newly admitted patient with bulging fontanels, setting-sun eyes, and lethargy.
Which of the following doctor’s order would you question? An order of:
*

1/1

a. Arterial blood draw


b. Magnetic resonance imaging
c. Computerized tomography scan
d. Lumbar puncture

2. A nurse observes a colleague performing a assessment of a child who has a head injury by
using the Glasgow coma scale. Which of the following assessments, if performed by the
colleague, indicates the colleague needs instructions regarding the use of this scale?
*

1/1

a. Motor responses
b. Deep tendon reflex

c. Verbal ability
d. Eye opening

3. A nurse would assess a patient who experiences prolonged vomiting for signs of
*

1/1
a. Hypovolemic shock

b. Metabolic acidosis
c. Water intoxication
d. Potassium excess

4. A psychiatric patient tells you, “I feel so depressed.” Which of these behavioral


manifestations would be the best documentation for the said cue?
*

1/1

a. Feels sad and alone, his heart is full of sorrow and pain
b. Cries when alone, avoids eye contact, with drooping posture

c. Doesn’t cooperate with the nurse, not having fun in going to group activities
d. Looking for the presence of nurse, interested in nurse-patient interaction

5. The nurse is handling a psychiatric patient. During one of their interactions, the patient told
the nurse, “I am mad.” How would the nurse document this?
*

0/1

a. Looks fierce and hostile


b. Clenched fists, ready to attack

c. Loud and belligerent, frown on face


d. Face looks angry, yet fearful
Correct answer
c. Loud and belligerent, frown on face

6. A student nurse asks why ink is imperative in documentation. This is because:


*

1/1

a. Changes will be easily-noticed, and the documents are permanent.


b. It looks more formal and readable compared to pencil.
c. Use of ink is according to the guidelines and policies by the hospital.
d. The other members of the health care team can understand the records better.

7. Computerized patient records may be used as legal documents. To maintain the legal
standards in computer records, which of the following should be observed?
*

0/1

a. Only the nurse involved in the care can see the patient records.

b. The nurse can get permission from the head nurse if a record has to be altered.
c. Once the data has been input, further changes will not be allowed.
d. The nurse counterchecks the computer input of another nurse if it is valid and appropriate.
Correct answer
c. Once the data has been input, further changes will not be allowed.

8. The family of a dependent psychiatric patient accuses Nurse Minda of negligence of her
nursing responsibilities. Nurse Minda uses the patient’s chart to:
*

1/1

a. Show that she in not the nurse in charge when negligence happened.
b. Serve as an evidence of Nurse Minda’s care given to the client at the time of the event in question

c. Prove the patient’s psychiatric illness and that his accusations are not based on reality.
d. Bring it to the court for Nurse Minda’s defense from patient’s statements

Situation
You are a nurse caring for patients who are victims of child abuse. The following questions apply:

9. A nurse found out that her patient in a medical ward was raped by her grandfather. She
appropriately reports this assessment to which of the following agencies/departments?
*

1/1

a. PNP
b. DSWD

c. Bantay Bata 163


d. Nearest child protection agency

10. The following are signs that child abuse is most likely present in a pediatric client except:
*

1/1

a. Abrasions on knees due to a bicycle crash as reported by the mother.

b. Child was brought to the clinic for burn injury that happened 4 days ago.
c. Mother tells that the bruises were due to home accident. The next day she tells that it’s due to a
fight with a neighbor.
d. Recurrent urinary tract infections

11. Which of the following is a characteristic of a sexual abuser?


*

1/1

a. Only males can be sexual abusers.


b. A sexual abuser is usually a stranger to the victim.
c. He was most likely sexually abused as a child.

d. He comes from a low-income family.

12. If child abuse is suspected in a family, which of the following approaches would a nurse
take when beginning to interview the child?
*

1/1

a. Speaking to the child by using specific, anatomically- correct terminology


b. Expressing concern to the child that something like this could have happened
c. Assuring the child that any information given will be kept confidential
d. Providing a private place to talk the child about the incident
13. A nurse is planning a community education presentation on domestic violence. Which of
the following factors should a nurse include?
*

1/1

a. Instructions on harmonious living with spouse


b. The telephone number of the local safe house

c. Ways to include the extended family


d. Assertiveness training

Situation
As a professional nurse you take into consideration the ethico-moral principles in providing nursing
care.

14. You released information over the phone to a caller who identified himself as the brother
of your patient. You found out later that the brother was out of town. Which of the following
rights did you violate?
*

0/1

a. Right to privacy
b. Right to continuity of care
c. Right to confidentiality

d. Right to respectful care


Correct answer
a. Right to privacy

15. Your patient is having difficulty making decision to undergo hip surgery. Which of the
following nursing actions BEST describes your advocacy roles as a nurse?
*

0/1

a. Protect patient’s autonomy and independence


b. Communicate patient’s needs to the interdisciplinary team
c. Advise the client to undergo surgery because it is best for her
d. Actively support patient’s decision

Correct answer
a. Protect patient’s autonomy and independence

16. Mrs Go needs to undergo a hip surgery. She refused to have it done even after the
attending surgeon has explained the procedure thoroughly. Which ethical principle applies in
this situation?
*

1/1

a. Autonomy

b. Justice
c. Non-maleficence
d. Beneficence

17. Informed consent is one of the patient’s bill of rights. One of its requirements is the
capacity of the patient to give it. Which of the following elements is/are related to this
capacity? (SELECT ALL THAT APPLY).
1. Patient is an adult
2. He is competent to make a choice
3. He has the freedom to make a choice
4. He can understand the consequences
*

1/1

a. 1, 2, 3
b. 1, 2, 4

c. 1, 2, 3, 4
d. 1, 3, 4

18. Another requirement of informed consent is voluntariness. It means, freedom of choice


without the following conditions, EXCEPT__________.
*

1/1

a. Force
b. Fraud
c. Consequences

d. Deceit

19. A nurse caring for a patient from a different culture notices that the patient did not eat the
food on the meal tray. Which of the following comments by the nurse demonstrate an
understanding of cultural diversity?
*

1/1

a. “What foods do you eat at home?”

b. “You need to eat to keep up your strength”


c. “You will lose weight if you do not eat.”
d. “Why didn’t you tell me you don’t like hospital food?’

20. Nurse Benedict is planning care with Jose. The client believes in “mal ojo” (the evil eye),
and uses treatment by a root healer. The nurse should do which of the following?
*

1/1

a. Avoid talking to the client about the root healer.


b. Explain to the client that Western medicine has a scientific, not mystical, basis.
c. Explain that such beliefs are superstitious and should be forgotten.
d. Involve the root healer in a consultation with the client, physician and nurse.

Situation
Marie is a new staff nurse in a psychiatric ward reviewing the guidelines on ethical principles.
21. The intervention by a psychiatric nurse that implements the ethical principle of autonomy
is when the nurse______________.
*

1/1

a. Stays with the client who is demonstrating high level of anxiety


b. Intervenes when a self-mutilating client attempts to slash wrist
c. Explores alternative solutions with the client, who later chooses one alternative

d. Suggest that two clients who are fighting be restricted to the unit

22. The nurse learns that a patient was admitted involuntarily on the shift. What assumption
can the nurse make about the patient?
*

1/1

a. For the first 48 hours, he can be given medication despite his objections
b. He can leave the ward upon demand
c. At the time of admission, he was considered to be an imminent danger to himself or to others

d. He has agreed to accept the treatment and participate fully in care planning

23. A male psychiatric nurse receives a call asking whether a certain person has been a
patient in the facility. How should the nurse respond? Nurse_______________.
*

1/1

a. States that he is unable to give any information to the caller

b. Asks the caller why the information is being sought


c. Suggests to the caller to speak to Mr X’s doctor
d. States that Mr X has been at the facility but gives no further information

24. A patient is about to receive electroconvulsive therapy (ECT) when the nurse sees that the
patient has not signed a consent for treatment. Which of the following facts should determine
the action of the nurse?
*
1/1

a. Verbal consent by the patient is sufficient


b. Another patient is needed to witness the consent form
c. Permission is granted by the patient when he signed the hospital’s admission form
d. Failure to obtain the patient’s written consent can result to a lawsuit

25. When the nurse is told by the patient that she consented to ECT out of fear of being
abandoned by her husband, what nursing action is required?
*

1/1

a. Explain that consenting to ECT will make her husband happy


b. Reassure the patient that her decision is sound
c. Document the patient’s statement

d. Reprimand the husband for coercing his wife

26. What other traits is expected from a client with paranoid personality disorder. The client:
*

1/1

a. Avoids responsibility for health care actions


b. Is afraid another person will inflict harm

c. Cannot follow limits set on behaviour


d. Depends on others to make important decisions

27. The client discusses current problems with the nurse. Which of the following
interventions should have priority in the nursing care plans. Have the client____________.
*

1/1

a. Discuss the use of defense mechanisms


b. Look at the source of frustration
c. Clarify his thoughts and beliefs about an event

d. Focus on the ways to interact with the others

28. You noticed that the client has impaired social skills. Which of the following short-term
goals is MOST appropriate for the client?
*

0/1

a. Address positive and negative feelings about self


b. Obtain feedback from other people

c. Identify personal feeling that hinder social interactions


d. Discuss anxiety-provoking situations
Correct answer
c. Identify personal feeling that hinder social interactions

29. The client makes an inappropriate and unreasonable report to you. Which of the following
principles of good communication skill is important for you to use?
*

1/1

a. Use nonverbal communication to address the issue


b. Use logic to address the client’s concerns
c. Tell the client that you do not share this interpretation

d. Confront the client about the stated misperception

30. Which of the following long-term goals is appropriate for this client?
*

0/1

a. Become appropriately interdependent with others

b. Become involved in activities that foster social relationships


c. Verbalized a realistic view of self
d. Take steps to address disorganized thinking
Correct answer
b. Become involved in activities that foster social relationships

Situation
Cathy is a staff nurse in a psychiatric ward of Hospital Chuprenia. The following questions apply:

31. The outcome that would be most appropriate for the patient who has a diagnosis of
agoraphobia would be that the patient will:
*

0/1

a. Go shopping in town
b. handle money without wearing gloves

c. Touch the neighbor’s dog


d. Bathe only once a day
Correct answer
a. Go shopping in town

32. Which of the following actions should a nurse take when making the first contact with a
paranoid patient?
*

1/1

a. Introduce self and avoid touching the patient

b. Avoid eye contact and shake hands with the patient.


c. Close the door to the interview room and remain standing
d. Wait for the patient to initiate communication

33. A male patient’s yearly laboratory screening reveals an elevated serum prostate-specific
antigen (PSA) level. To which of the following nursing diagnosis should a nurse give priority
for this patient?
*

0/1
a. Defensive coping

b. Hopelessness
c. Anxiety
d. Social Isolation
Correct answer
c. Anxiety

34. A schizophrenic patient says to a nurse, “You are wearing a pretty red dress. Tomatoes
are red. Vegetables make you healthy. I am not healthy.” A nurse should recognize that these
statements are an example of
*

1/1

a. Echolalia
b. Neologisms
c. Confabulation
d. Looseness of association

35. Which of the following sets of symptoms is characteristic of generalized anxiety


disorder?
*

1/1

a. Uncontrollable worrying, significant distress or impaired social functioning for at least 6 months.

b. Intense fear and helplessness within 1 month after exposure to traumatic event that lasts 2 days to 4
weeks.
c. Re-experiencing of an extremely traumatic event and numbing of responsiveness within 3 months
to years after the event.
d. Significant anxiety provoked by a specific feared object or situation.

36. Obsessive-compulsive disorder is characterized by recurrent, unwanted thoughts which


lead to ritualistic and repetitive behaviors in an attempt to neutralize anxiety. Which of the
following medications are appropriate for OCD?
*
1/1

a. Tacrine (Cognex)
b. Bupropion (Wellbutrin)
c. Haloperidol (Haldol)
d. Clomipramine (Anafranil)

Situation
You are a staff nurse in a government hospital being transferred to the Psychiatric Unit. You were
required to equip yourself by attending the enhancement program on Crisis Intervention. To assess
your knowledge and skills on the subject you were given a pre-test.

37. A crisis that is acute but temporary and due to external source is__________.
*

0/1

a. Developmental
b. Transitional

c. Traumatic
d. Dispositional
Correct answer
d. Dispositional

38. Which of the following nursing interventions is the most appropriate for a client who is in
the early state of crisis?
*

0/1

a. Encourage client to express feeling and emotions related to crisis


b. Require client to be actively involved in establishing goals
c. Encourage client to begin the development of insight

d. Ask client to evaluate the situation


Correct answer
a. Encourage client to express feeling and emotions related to crisis
39. Which of the following is NOT an assumption in the concept of crisis?
*

1/1

a. Crisis is acute and resolved within a short period of time


b. All individuals experience a crisis
c. Crisis is a growth-retarding factor to the emotional development of a person

d. Specific identifiable events precipitate a crisis

40. The MAIN objective of crisis intervention is to_____________


*

0/1

a. Make the person realize his/her mistakes


b. Ensure patient’s safety

c. Return the person to the root of the crisis to identify the cause
d. Eliminate the stressor
Correct answer
c. Return the person to the root of the crisis to identify the cause

Situation
Nurse Katrina wants to improve in her care for patient EJ, who is an alcoholic. She asks help from her
Head Nurse Chantele

41. Katrina goes to Mar's bedside to greet him. Chantele corrects Katrina of her greeting
which is NOT appropriate to EJ?
*

1/1

a. "Hi, EJ!„ so you got drunk last night"

b. "Hi, EJ!, I heard you enjoyed yourself last night"


c. "Hi, EJ!, I heard you had a drinking spree last night"
d. "Hi, EJ!, How was your drinking affair last night"
42. EJ turned his back away from Nurse Katrina, saying "It's none of your business, you ugly
duckling." The appropriate response of Nurse Katrina would be:
*

1/1

a. "What you said hurt me, you alcoholic!"


b. "You beast, you are as ugly as I am"
c. "You really arc a drunkard"
d. "I don't think you mean what you have just said, do you?"

Situation
Tommy is a 7-year old, Grade one kid, who is diagnosed with Attention Deficit/ Hyperactive Disorder
(AD/HD).

43. Manifestations of ADHD are usually identified in what situation or event?. When a child
is
*

0/1

a. At home, by parents
b. With peers, during play

c. Enrolled in the education system


d. Is brought to a well baby clinic
Correct answer
c. Enrolled in the education system

44. The school nurse reports that Tommy frequently exhibits the following behavioral
manifestations of ADHD, except:
*

0/1

a. Interrupt others and can't take turns


b. Moody and bad-tempered behavior
c. Easily distracted and forgetful
d. Incorrect and messy work
Correct answer
d. Incorrect and messy work

45. Tommy tells the nurse, "I don't have friends because I'm stupid." Which of the following
nursing diagnosis would the nurse identify for him?
*

1/1

a. Ineffective coping
b. Anxiety
c. withdrawal syndrome
d. low self esteem

46. The nurse expects Tommy to have impaired social interaction as the child exhibits
excessive talking, short attention span, and low frustration tolerance. Which of the following
nursing interventions is NOT useful in his case?
*

1/1

a. Call Tommy's name and establish eye contact


b. Give instructions to the child slowly using simple language
c. Provide positive feedback for completion of each task
d. Give complex tasks one at a time

Situation
Franky Omeley is an 11-year old Grade IV pupil in a private school was diagnosed to have an
attention-deficit hyperactivity disorder (ADHD).

47. Which of the following is NOT a MAIN characteristic of ADHD?


*

1/1

a. Passivity
b. Inattention
c. Hyperactivity
d. impulsivity

48. His teacher has observed Franky Omeley to have difficulty waiting for his tutor often
batting into conversations and blurts out answers without waiting for questions to be finished.
These are manifestations of_________.
*

1/1

a. Hyperactivity
b. Inattention
c. passivity
d. impulsivity

Situation
You are a staff nurse in the Orthopedic Unit of the Department of Surgery of the Hospital trying to do
research studies.

49. You are assisting in a research study on assessing patient's reactions to the use of new
dressing material. A medical student questions the credibility of the nursing research. Your
response would be:
*

0/1

a. To keep quiet
b. "Nursing research is essential for the development of nursing science"
c. "Doing research is one of the competencies of professional nurses"

d. "Nursing practice is based on research findings"


Correct answer
b. "Nursing research is essential for the development of nursing science"

50. Which of the following research methods has for its purpose to describe social processes
present within human interactions?
*
0/1

a. Phenomenology

b. Grounded theory
c. Participatory action research
d. Case study
Correct answer
c. Participatory action research

51. In a clinical question, "Is breastfeeding more effective in increasing the birth weight of
preterm infant than adding corn oil to the infant formula?" what is the intervention of
interest?
*

0/1

a. Infant feeding
b. Increasing the birth weight
c. Preterm infant
d. Adding corn oil to the infant formula

Correct answer
a. Infant feeding

52. In a research question, "What is the relationship between wound healing and nutrition
among elderly patient with hip surgery?" which one is the dependent variable?
*

1/1

a. Elderly patient
b. Wound healing

c. hip surgery
d. Nutrition

Situation
You are a nurse in the psychiatric ward caring for several patients.

53. The nurse provides information to a depressed patient and his family about
electroconvulsive therapy (ECT). Which of the following statements would the nurse include
in the teaching?
*

1/1

a. “The patient will have a minimal muscle twitching during the treatment.”
b. The patient must be in restraints following the treatments.”
c. “The patient will remain awake and alert during the treatment.”
d. “The patient must remain flat on his back for one hour after treatment.”

54. Neuroleptic malignant syndrome (NMS) is potentially lethal complication of treatment


with antipsychotic drugs. Which of the following manifestations should the nurse recognize
as an early sign of neuroleptic malignant syndrome?
*

1/1

a. Difficulty swallowing
b. Delirium
c. Muscle stiffness

d. Respiratory depression

55. Jose spends most of the day lying in bed with the sheet pulled over his head. Which of the
following approaches by Nurse Benedict is most therapeutic?
*

1/1

a. Wait for the client to begin the conversation.


b. Initiate contact with the client frequently.

c. Sit outside the client’s room.


d. Question the client until he responds.
56. The client exhibits a flat affect, psychomotor retardation, and depressed mood. The nurse
attempts to engage the client in an interaction but the client does not respond to the nurse.
Which response by the nurse is most appropriate?
*

1/1

a. “I’ll sit here with you for 15 minutes.”

b. “I’ll come back a little bit later to talk.”


c. “I’ll find someone else for you to talk with.”
d. “I’ll get you something to read.”

57. A patient express many physical complaints during the first two weeks on the alcohol
rehabilitation unit. The result of the physical examinations have been negative. The patient
frequently approaches staff members to request for medication for her discomfort. Based on
the patient’s members to request for medication for her discomfort. Based on the patient’s
behavior, which of the following interpretations is correct?
*

1/1

a. The patient is trying to make the staff feel guilty.


b. The patient is attempting to relieve the anxiety.

c. The patient is experiencing organic pain from alcohol withdrawal.


d. The patient is using a more mature way of meeting her needs than alcohol.

Situation
Terrence Romeo is a 21-year old Volleyball varsity player in a leading university. He was admitted
for eye trauma after a Volleyball game.

58. Terrence has a bluish discoloration in his right eye. This condition is caused by:
*

1/1

a. skin irritation
b. the impact of the blow
c. tissue swelling around the eye
d. rupture of a blood vessel in the conjunctiva

59. A potential result of blunt eye trauma is bleeding into the anterior chamber if the eye.
This condition is termed of:
*

1/1

a. Chemosis
b. Hyphemia

c. conjunctival hemorrhage
d. ecchymosis

60. Which of the following is the PRIORITY nursing diagnosis for a patient with eye trauma?
*

1/1

a. Impaired visual acuity


b. Risk for eye injury: ocular
c. Risk for eye injury: visual
d. Impaired tissue integrity: ocular

61. When teaching Terrence in relation to eye injuries, which of the following would be your
focus?
*

0/1

a. Instillation of eye drops


b. Prompt treatment of the injury
c. Prevention and first aid measures
d. Use of protective devices

Correct answer
b. Prompt treatment of the injury
62. Another cause of eye trauma is a chemical burn. Which of the following measures would
you advise when it happens in the home?
*

0/1

a. Apply eye drops to the affected side


b. Take the victim to the nearest hospital solution
c. Irrigate the affected eye with normal saline

d. Irrigate the affected eye with tap water


Correct answer
d. Irrigate the affected eye with tap water

Situation
You are an advocate for women's rights. You were invited as a resource speaker on "Violence against
women and their children." the following questions relate to this.

63. Women and their children are protected against violence under this law:
*

1/1

a. R. A. 7610
b. R.A. 8353
c. R.A. 7160
d. R.A. 9262

64. One of the violent acts is sexual violence. Which of the following does NOT constitute
sexual violence?
*

1/1

a. Sexual Abuse
b. Incest
c. Economic abuse
d. Drug-facilitated sexual assault

65. The health worker's Main concern when abused, maltreatment or neglect is suspected
should be:
*

1/1

a. Verbalization of feelings
b. Airway
c. Reporting to the legal authorities
d. safety and welfare of the client

66. Which of the following is the most common clinical manifestation of a child neglect?
*

1/1

a. Malnutrition and dehydration

b. Cough and colds


c. Frequent urinary tract infection
d. Crying incessantly

67. Which of the following should be the focus of care for client who are suspected to be
abused, maltreated or neglected?
*

0/1

a. Proper documentation

b. Immediate treatment of any injury


c. Verbalization of feelings
d. Preservation of evidence
Correct answer
b. Immediate treatment of any injury

Situation
Nurse Camille is a nurse for several clients in the National Center for Mental Health. The following
questions apply:

68. A patient has just attempted to commit suicide by hanging. To prevent the patient from
attempting this again, which of the following is appropriate for the nurse to do?
*

0/1

a. Ask a nurse to watch the patient constantly.


b. Take him to seclusion room.
c. Ask the peer to safeguard client

d. Remove one’s clothing to make sure no other items are with the patient.
Correct answer
a. Ask a nurse to watch the patient constantly.

69. A client with a diagnosis of antisocial personality disorder was given a 2-hour pass to
leave the hospital. The client returned to the unit 15 minutes past curfew and did not sign in.
The next day, this behavior is brought up in a group meeting. The client says, "It's all the
nurse's fault. The nurse was right there and did not remind me to sign in." What is the best
response by the nurse?
*

1/1

a. “I'm sorry. I should have reminded you to sign in."


b. “It is not my fault that you forgot to sign in."
c. “It is your responsibility to sign in when you return from a pass."

d. “You were late coming back from your pass. Is that why you did not sign in?"

70. A patient is experiencing acute dystonic reaction. All of the following PRN medications
are prescribed. Which of the following medication should the nurse administer?
*

0/1

a. Chlorpromazine hydrochloride (Thorazine)


b. Lorazepam (Ativan)
c. Diphenhydramine Hydrochloride (Benadryl)
d. Alprazolam (Xanax)
Correct answer
b. Lorazepam (Ativan)

Situation
A nurse is attempting other biobehavioral nursing interventions when taking care of her patients.

71. Use of touch as part of nursing care for geriatric clients is generally considered as
therapeutic. Which of the following is true about use of touch?
*

1/1

a. It is calming for the elderly patient.


b. Most elderly patients get angry when touched.
c. Its effect is based on the individual preferences.

d. Patient’s face is touched to communicate empathy most effectively.

72. Time travel is________:


*

0/1

a. To remember the pain when the pain subsides


b. Through deep meditative techniques and imaginations, divert using your mind thinking place
you've never been into.

c. Through hypnosis as well, you can absolutely go back into your past and heal things.
d. Ability that allows humans to be constantly aware of the past and the future and not only for
recollection of past personal experiences
Correct answer
d. Ability that allows humans to be constantly aware of the past and the future and not only for
recollection of past personal experiences

Situation
Glenda is a married female who was admitted to the health psychiatric unit because of substance
abuse.
73. Which of these initial assessments would indicate presence of withdrawal?
*

1/1

a. Elevated vital signs and nervousness

b. Drowsiness
c. Hypotension, bradycardia
d. Apathy

74. Which of these medications will manage alcohol withdrawal?


*

1/1

a. Methadone
b. Disulfiram
c. Naloxone
d. Lorazepam

75. Glenda’s spouse comes to you to tell you that he has to care for Glenda because the
patient is in denial of her substance abuse issues. A nursing diagnosis was made:
Dysfunctional family processes: Alcoholism r/t codependency, as evidenced by the spouse’s
rescuing behavior. Which goal would indicate an understanding of the current situation and
the connection between the diagnosis and outcome?
*

0/1

a. Glenda’s spouse admits codependency


b. Glenda’s admission of being an alcoholic
c. Raising a considerable amount of funds for Glenda’s treatment
d. Family achieves improved family dynamics

Correct answer
b. Glenda’s admission of being an alcoholic
76. Glenda was admitted to the facility to treat her alcohol treatment. This was insisted by the
spouse. During interview, Glenda tells you, “I may drink everyday but my habit is controlled
and non-destructive.” You recognize this as denial of the current situation. How would you
best respond?
*

1/1

a. “What have been the negative consequences of your drinking?”

b. “You are not telling the truth, I suppose.”


c. “Is it okay if I will ask your spouse to observe and note your pattern of drinking?”
d. “How much do you drink to say that it’s controlled?”

77. Glenda has completed one week of treatment for her alcohol dependency. She exclaims,
“I will not resort to alcohol drinking now, but there seems nothing left for me.” How would
you best respond?
*

0/1

a. “I am here whenever you needed me. I’m your nurse.”

b. “You still have your family who is always there for you.”
c. “I will give you this prescribed medication to help you with the withdrawal.”
d. “What did your addiction mean to you emotionally?”
Correct answer
d. “What did your addiction mean to you emotionally?”

78. What drug is given to maintain abstinence from opiates?


*

1/1

a. Disulfiram (Antabuse)
b. Methadone (Dolophine)

c. Lorazepam (Ativan)
d. Naloxone (Narcan)
79. Which of the following suggestions should a nurse make to a known poly substance-
abusing woman who is 18 weeks pregnant?
*

0/1

a. “If you cannot stop taking drugs, you might consider terminating the pregnancy.”
b. “You should stop using all drugs immediately before your baby develops birth defects.”
c. “If you enter a drug treatment program now, your baby will be born healthy.”
d.”It may not be possible for you to stop drugs completely, but you should consider limiting the drugs
you use during pregnancy.”

Correct answer
b. “You should stop using all drugs immediately before your baby develops birth defects.”

Situation
A nurse in San Bernardino Hospital is caring for an HIV-positive patient.

80. The patient is prescribed with Zidovudine (Retrovir). What should the nurse instruct the
patient to do?
*

0/1

a. “Take the medication only with food.”

b. “Lie down after taking capsules.”


c. “Instruct that with proper usage, it can cure the disease.”
d. “Take the drug at specified intervals.”
Correct answer
d. “Take the drug at specified intervals.”

81. What is the major problem the nurse should be cautious of when taking care of a patient
taking Zidovudine
*

1/1

a. The details of the prescription


b. Drug-drug interactions
c. The side effects in taking the drug

d. Patient education

82. Which among the following laboratory findings is the MOST significant in an HIV-
positive patient?
*

1/1

a. Decreased CD4+ / T-helper cell count

b. Decreased RBCs
c. Increased AST/ALT
d. Decreased WBCs

83. Headache, impaired cognition, and loss of vision are early manifestations of which of the
following?
*

1/1

a. Acquired immunodeficiency syndrome (AIDS)

b. Severe Combined Immunodeficiency (SCID)


c. Systemic inflammatory response syndrome (SIRS)
d. Multi-acquired agranulocytosis-induced modulation syndrome (MAAMS)

84. The nurse came up with the nursing problem “Imbalanced Nutrition: Less than body
requirements”. Her intervention for this would focus on:
*

0/1

a. Encourage to drink water frequently.


b. Health teaching on nutritious food that must be consumed.
c. Ask the patient to consume the serving of food from the dietary department.

d. Allowing the patient to have small meals evenly spread throughout the day
Correct answer
d. Allowing the patient to have small meals evenly spread throughout the day

Situation
Marilou, a head nurse in the OB ward is managing her staff and personnel. She must be guided by the
principles of leadership and management. The following questions apply:

85. The nurse entered a patient’s room to perform tepid sponge bath. The patient refused the
attempt but the nurse responded, “I need to bathe you or else I will have to tie you down.”
The nurse has committed what tort?
*

0/1

a. Assault
b. Battery
c. False imprisonment

d. Invasion of privacy
Correct answer
a. Assault

86. During a meeting with the staff, the head nurse involves the group in decision making
regarding the scheduling of ward classes. The group was divided into two decisions. What
must the head nurse do?
*

0/1

a. Agree on a 60-40% rule.


b. The head nurse takes over and decides for the group.
c. Come up with a majority vote.
d. Ask the two groups to meet halfway.

Correct answer
c. Come up with a majority vote.

87. According to RA 9173, the following are the guidelines to be observed by registered
nurses, except:
*
0/1

a. Preserve life, respect human rights, and promote health environment at all times.
b. Be equipped with the knowledge of health resources within the community, and take active roles in
primary health care.
c. Actively participate in programs, projects, and activities that respond to the problems of the society.
d. Project an image that will uplift the nursing profession at all times.

Correct answer
a. Preserve life, respect human rights, and promote health environment at all times.

88. The head nurse is starting quality improvement in their ward using the Six Sigma
approach. What is the proper sequence to follow?
i. Analyze
ii. Define
iii. Control
iv. Measure
v. Improve
*

0/1

a. ii, i, iii, iv, v

b. iv, ii, iii, i, v


c. ii, iv, i, v, iii
d. iv, ii, i, v, iii.
Correct answer
c. ii, iv, i, v, iii

89. ISBAR is a verbal communication tool that provides structured, orderly approach in
providing accurate, relevant information in emergent patient situations and handoffs. Which
of the following is not parts of ISBAR?
*

1/1

a. Situation
b. Background
c. Analysis
d. Recommendation

Situation
A nurse in the hospital is teaching a new staff nurse about the guiding principles and policies of
PhilHealth.

90. Which of the following are not included in the mandate and functions of PhilHealth?
*

0/1

a. Use services now, pay later.


b. Collect and disburse National Health Insurance Fund.
c. Rich subsidizes the poor.
d. To receive and manage donations.

Correct answer
c. Rich subsidizes the poor.

91. The following diseases treated at the outpatient department are covered by PhilHealth
except:
*

1/1

a. Cardiac diseases

b. Tuberculosis
c. Renal failure
d. Cancer

92. Which of the following PhilHealth benefit packages is not included in the case payment
scheme?
*

0/1

a. Newborn Care Package (NCP)


b. Pneumonia I (Low risk)
c. Outpatient HIV/AIDS Treatment Package
d. Asthma

Correct answer
b. Pneumonia I (Low risk)

Situation
You are the nurse in the emergency room. A 22 year old male is brought in with an apparent head
injury after being involved in a serious car accident. He is unconscious on arrival and exhibits signs of
increasing intracranial pressure.

93. When the client arrives at the ER, which of the following should you consider a priority
care?
*

1/1

a. Replace blood loss.


b. Determine if he has a fracture in the neck.
c. Establish an airway.

d. Stop bleeding from the open head wound.

94. You continue to assess the client. Which of the following respiratory signs would indicate
increasing intracranial pressure in the brain stem?
*

1/1

a. Rapid, shallow respiration.


b. Asymmetric chest excursion.
c. Nasal flaring.
d. Slow, irregular respiration.

95. You assess the client frequently for signs of increasing intracranial pressure which is
______.
*

1/1
a. Decreasing systolic pressure.
b. Decreasing body temperature.
c. Tachycardia
d. Unequal pupil size.

96. You position the client. Which position would be MOST appropriate?
*

1/1

a. Elevate the head to 30 to 45 degrees.

b. Elevate the head to two pillows.


c. Place the client in a Trendelenburg position
d. Place the client in left Sim’s position.

97. You are aware that early indication of deterioration in the neurologic status of client is a
_________.
*

1/1

a. Dilated, fixed pupil.


b. Widening or pulse pressure.
c. Decrease in the level of consciousness

d. Decrease in the pulse rate.

Situation
Glenda is a married female who was admitted to the health psychiatric unit because of substance
abuse.

98. Which of these initial assessments would indicate presence of withdrawal?


*

1/1

a. Elevated vital signs and nervousness


b. Drowsiness
c. Hypotension, bradycardia
d. Apathy

99. Which of these medications will manage alcohol withdrawal?


*

1/1

a. Methadone
b. Disulfiram
c. Naloxone
d. Lorazepam

100. Glenda’s spouse comes to you to tell you that he has to care for Glenda because the
patient is in denial of her substance abuse issues. A nursing diagnosis was made:
Dysfunctional family processes: Alcoholism r/t codependency, as evidenced by the spouse’s
rescuing behavior. Which goal would indicate an understanding of the current situation and
the connection between the diagnosis and outcome.
*

0/1

a. Glenda’s spouse admits codependency


b. Glenda’s admission of being an alcoholic
c. Raising a considerable amount of funds for Glenda’s treatment
d. Family achieves improved family dynamics

Correct answer
b. Glenda’s admission of being an alcoholic

You might also like